Sie sind auf Seite 1von 66

Visitors to the park have often looked up into the leafy canopy and saw monkeys sleeping on the

branches, whose arms and legs hang like socks on a clothesline.

(A) saw monkeys sleeping on the branches, whose arms and legs hang
(B) saw monkeys sleeping on the branches, whose arms and legs were hanging
(C) saw monkeys sleeping on the branches, with arms and legs hanging
(D) seen monkeys sleeping on the branches, with arms and legs hanging
(E) seen monkeys sleeping on the branches, whose arms and legs have hung

Whose can incorrectly refer to branches also and not sure if it refers to monkeys.

-------------------------------------------------------------

Many children of illegal immigrants grow up having to keep their status a secret, heavy
psychological burden.

A. a secret, heavy
B. a secret and a heavy
C. a secret, a heavy
D. secret, with a
E. secret, which is a

Here, a psychological heavy burden is an appositive that modifies the theme of having to keep
their status a secret. The phrase' having to keep their status a secret is a gerund (in effect a
noun) acting as an object noun of the verb grow up.
In sum, whether something is an appositive or not can be seen from the distinct fact that it is a
noun.

Choice A illogically attempts to describe the noun that children are keeping as one entity — a
secret, heavy burden. Choice E makes a similar mistake, as the word "which" sets up a modifier
that would be defining a secret as a heavy psychological burden.

Choices B and D also commit similar errors: by connecting "secret" and "a heavy psychological
burden" with a conjunction, each choice seems to view the two items as separate and parallel
(students have to keep two things, a secret and a burden). Because this is an illogical meaning,
both choices are incorrect.

As per my understanding the difference between A and C is adding an article makes the phrase
after comma a noun phrase or absolute modifier. 

Correct ans is C.
The height of handle of the cricket bat is one-third the total length, creating a sweet spot at the
lower end of the bat, but now per the new design it is constructed  such that the sweet spot
spans across its entire length.

A. The height of handle of the cricket bat is one-third the total length, creating a sweet spot at
the lower end of the bat, but now per the new design it is constructed

B. Per the current design, the height of handle of the cricket bat is one-third the total length and
it creates a sweet spot at the lower end of the bat, but now per the new design it is constructed

C. Previously designed such that the height of its handle was one-third its total length, was
creating a sweet spot at its lower end, the cricket bat is now constructed

D. Previously designed such that the height of its handle was one-third its total length, creating a
sweet spot at its lower end, the cricket bat is now constructed

E. Previously designed such that the height of its handle was one-third its total length, creating a
sweet spot at its lower end, the cricket bat now constructed

The height of handle of the cricket bat is one-third the total length, creating a sweet
spot at the lower end of the bat, but now per the new design it is constructed  such
that the sweet spot spans across its entire length.
A.The height of handle of the cricket bat is one-third the total length, creating a
sweet spot at the lower end of the bat, but now per the new design it is constructed -
Unnecessary lengthy & fails to convey the meaning. Incorrect
B.Per the current design, the height of handle of the cricket bat is one-third the total
length and it creates a sweet spot at the lower end of the bat, but now per the new
design it is constructed - Lacks parallelism in the the 2nd highlighted portion as
Design is followed by Bat in 2nd part but Height in the 1st part. Incorrect.
C.Previously designed such that the height of its handle was one-third its total
length, was creating a sweet spot at its lower end, the cricket bat is now
constructed - Use of WAS before a modifier creates a Run-on. Incorrect.
D.Previously designed such that the height of its handle was one-third its total
length, creating a sweet spot at its lower end, the cricket bat is now constructed -
Absolutely Correct
E.Previously designed such that the height of its handle was one-third its total
length, creating a sweet spot at its lower end, the cricket bat now constructed -
Initial modifier is absolutely correct but the Independent clause lacks Verb, creating
a Fragment. Incorrect.

whenever you have a construction like this:


1) clause1(sub1+verb), but clause 2
2)clause1(sub1+verb), and clause 2
3)although clause 1(sub1+verb), clause 2

and if in clause 2 the subject is a PRONOUN then its antecedent is always the SUBJECT OF
CLAUSE 1.

The height of handle of the cricket bat is one-third the total length, creating a sweet spot at the
lower end of the bat, but now per the new design it is constructed .......................

SUBJECT -VERB==>HIGHLITED IN GREEN


MODIFIER HIGHLITED IN RED
Scientists are investigating the possibility of microbial life in Lake Vostok, a huge
body of water trapped beneath the ice of Antarctica; the lake's ecosystem is one that
they believe is very similar to that beneath the frozen surface of Jupiter's moon
Europa.

Choices:
(A) that they believe is
(B) that they believe it to be
(C) they believe it is
(D) they believe that is
(E) they believe to be

Answer E - The issue here is the modifier "that they believe …". The proper idiomatic form is
"they believe to be"; therefore, the correct answer is (E).

A vertical scan reveals the two basic choices "that they" versus "they". While "that" is not
incorrect per se, here it is unnecessary because the word "one" sufficiently introduces the
modifier. Thus we eliminate (A) and (B).

(C) contains the word "it", which is redundant because of the word "one" which appears
previously.

(D) is unidiomatic, since the proper form is "X they believe to be Y", not "X they believe that is
Y". Therefore we eliminate (D), and thus choice (E) is correct. Note that this sentence (even with
the correct answer choice) is not necessarily the most elegant or efficient way to communicate
the intended meaning. Do not let this bother you. Focus on the specific issue being tested. The
underlined portion is your best clue as to what's being tested, though the non-underlined portion
usually has an impact on the issue being tested as well. On Test Day you may see some
Sentence Correction questions for which you could construct a better sentence than any offered,
but thinking that way will not increase your GMAT score. Always focus on what is being tested in
the Sentence Correction question on the screen in front of you.

You should use THAT when you have to define or provide important information about a noun.
Without that information you cannot identify the noun. For example:
I want to apply to a business school THAT ranks as one of the best schools in the world.

But if you can identify the noun, THAT is not required. In that case, you could use ", which". For
example:

I want to apply to Harvard Business School, WHICH ranks as one of the best schools in the
world.

You can see that "ranks....world" is just additional information. If you read only "I want to apply
to Harvard Business School", you can identify the school. More information is not required.

Now let's see the question:

If we only read "the lake's ecosystem is one", we can see that it would be wrong. "ONE" has to
be defined; we need THAT.

But, why can option E be correct if it doesn't have THAT? And here comes the trick:

We can ommit THAT when the object of the verb of the clause that defines (in this case, "they
believe...") is the noun that it is being defined.
-------------------------------
Studies show that young people with higher-than-average blood pressure and their families have
a history of high blood pressure are more likely than others to develop a severe form of the
condition.

(A) and their families have a history of high blood pressure

(B) whose families have a history of high blood pressure

(C) and a history of high blood pressure runs in the family

(D) whose families have a history of high blood pressure running in them

(E) with a history of high blood pressure running in their family

Studies show that young people (with higher-than-average blood pressure) (whose families


have a history of high blood pressure) are more likely than others to develop a severe form of
the condition.

as you can see, "with higher....." is a discriptive phrase describing "young people," then right
after that, we have another descriptive phrase that runs immediately after it (whose
families.....). Because it is perfectly fine to have 2 phrases running after each other, the "whose
families" is also describing the same noun "young people." If there was also a third phrase that
would come right after "whose families.....", THEN we MUST have a comma in order to avoid any
reference confusion. However, according to the grammar rule, it is perfectly fine to have two
descriptive phrases running after each other like our problem above.

I really think it's about being logical about it. Since we know about the rule that 2 descriptive
phrases can run after each other, if you think that the second phrase is describing the first
phrase, then take a look to see if the connection between the 2 phrases is logical. If the
connection isn't logical, then it must be referring to the noun before the first phrase. As long as
the grammar rule isn't broken, then it should be acceptable. On the other hand, if the 2 phrases
could logically describe each other, but this is not the intension of the sentence, then a comma
in this case would be necessary.

Studies show that young people with higher-than-average blood pressure and with a history of
high blood pressure running in their family are more likely than others to develop a severe form
of the condition.

The answer will still not be correct because the problem of “running” still persists. Also “and”
introduces a list in the sentence. The sentence now talks about two kinds of young people – first
kind with higher-than-average blood pressure and the second kind with the history of blood
pressure in the family. This certainly is not the intended meaning.

@macjas:
The correct “idiom” is “run/running in the family” alright. But when we use the term history, we
say that “he has a history of heart disease in his family”. We do not use “run/running” with
history. We can say: Heart diseases run in his family, OR His family has the history of heart
diseases.
(A) and their families have a history of high blood pressure ; The choice is trying to connect a
phrase and an IC (the Main Clause) with a co-ordinating conjunction : one can join only two
parallel factors with “and”

(B) whose families have a history of high blood pressure; correct choice; whose modifies the
young people rightly, the intervening prepositional phrase is an essential modifier. Please note
that pressure cannot have families. Hence, it is logical that whose modifies the somewhat distant
people, rather than the proximate but singular pressure.

(C) and a history of high blood pressure runs in the family; same error as in A

(D) whose families have a history of high blood pressure running in; ‘them’ is ambiguous. Does it
refer to families or peoole?

(E) with a history of high blood pressure running in their family; The most glaring bad logic is to
say that all the young people have one family rather than families.

(B) whose families have a history of high blood pressure: Correct. Notice that “whose” is a
relative pronoun, a noun modifier that is modifying a little far away noun “young people”. This is
so because the prepositional phrase preceding “whose” cannot be placed anywhere else in the
sentence without affecting the meaning. Hence, the relative pronoun can jump over “with
higher-than-average blood pressure” to modify “young people”.

(D) whose families have a history of high blood pressure running in them: Incorrect. The verb-
ing modifier “running” is modifying the preceding noun entity “history of high blood pressure”,
meaning that the “history of high blood pressure” run in the “young people and their family”.
This is illogical.

(E) with a history of high blood pressure running in their family: Incorrect. Use of “running”
seems to suggest that “a history” runs in their family. This modification does not make sense.

Although just inside the orbit of Jupiter, amateur astronomers with good telescopes should be
able to see the comet within the next few weeks.

(A) Although just inside the orbit of


(B) Although it is just inside the orbit of
(C) Just inside the orbit of
(D) Orbiting just inside
(E) Having orbited just inside

What's special about (B)? Well, as mentioned earlier, (B) includes the infamous "it." We
don't really know what "it" is referring to until later in the sentence when "comet" is mentioned.
But (B) is the only answer choice with the "it" that makes it a better answer choice than all the
other ones.
A. Although just inside the orbit of jupiter
This is a phrase and should modify the word immediately following comma.
(B) Although it is just inside the orbit of jupiter
This is a dependent clause..so need not modify the word following comma.
Eg: Although they were under the radar until recently, Joule Unlimited, the firm in Mass has two
issued and at least three pending U.S. Patents.
(C) Just inside the orbit of
Same as A
(D) Orbiting just inside
Same as A
(E) Having orbited just inside
Same as A

Unlike Mozart, whose funeral was sparsely attended being buried in an unmarked common
grave, Beethoven’s was attended by more than thirty-thousand mourners and his final resting
place lies in a famous Vienna cemetery near the graves of Schubert and Brahms.

A. being buried in an unmarked common grave, Beethoven’s was attended by about thirty
thousand mourners and his final resting place lies in a famous Vienna cemetery near the graves
of Schubert and Brahms

B. being buried in an unmarked common grave, Beethoven’s funeral was attended by about
thirty-thousand mourners and his final resting place was near the graves of Schubert and
Brahms in a famous Vienna cemetery

C. and he was buried in an unmarked common grave, Beethoven was given a funeral attended
by more than thirty-thousand mourners and his final resting place lies near the graves of
Schubert and Brahms in a famous Vienna cemetery

D. and who was buried in an unmarked common grave, Beethoven’s funeral was attended by
more than thirty-thousand mourners and his final resting place lies in a famous Vienna cemetery
near the graves of Schubert and Brahms

E. and who was buried in an unmarked common grave, Beethoven was given a funeral attended
by more than thirty-thousand mourners and lies buried near the graves of Schubert and Brahms
in a famous Vienna cemetery

This sentence has four errors. First, the subordinate clause “whose funeral was
sparsely attended being buried in an unmarked communal grave” is awkward
grammatically and ambiguous in meaning; the clause could be interpreted to mean
that the funeral was sparsely attended at the time Mozart was being buried, or it could
be interpreted to mean that the funeral was sparsely attended because Mozart was
buried in an unmarked grave. Second, it is not clear whether the possessive
“Beethoven’s” refers to “funeral” or to “grave.” Third, the sentence makes an illogical
comparison between Mozart and either Beethoven’s funeral or Beethoven's grave.
Finally, the prepositional phrase “near the graves of Schubert and Brahms” is a
misplaced modifier. Since it immediately follows “cemetery,” it appears to describe the
location of the cemetery rather than that of the grave.
(A) This choice is incorrect since it repeats the original sentence.
(B) First, the subordinate clause “whose funeral was sparsely attended being buried
in an unmarked communal grave” is grammatically awkward and ambiguous in
meaning. Second, while the introduction of “funeral” makes the possessive
“Beethoven’s” unambiguous, the sentence still illogically compares “Mozart” to
“Beethoven’s funeral.” Finally, the past tense is used illogically in the clause “his final
resting place was ….” Since this clause discusses Beethoven’s final resting place, it
describes a state of being that is still true today; hence, the use of the present tense
is appropriate to describe where Beethoven’s body currently lies.
(C) “Mozart” is followed by two clauses, “whose funeral was sparsely attended” and
“he was buried in an unmarked communal grave.” The second of these clauses is
incorrect because: 1) it should be a subordinate clause modifying Mozart, and should
therefore start with "who was buried", and 2) it should be parallel to the first clause,
and should therefore start with "who was buried", and 3) it should not make the
illogical assertion that “Unlike Mozart, he [Mozart] was buried….”.
(D) This choice makes an illogical comparison between "Mozart" and "Beethoven’s
funeral." In addition, the prepositional phrase “near the graves of Schubert and
Brahms” is a misplaced modifier. Since it immediately follows “cemetery,” it appears
to describe the location of the cemetery rather than that of the grave.
(E) CORRECT. “Mozart” is now modified by two subordinate clauses, “whose funeral
was attended….” and “who was buried ….,” each properly introduced by the relative
pronouns “whose” and “who” respectively. In addition, “Mozart” is now logically
compared to “Beethoven.” Finally, the phrase “near the graves of …” unambiguously
modifies “lies buried.”

Also, Lay takes the verb form directly after ir as a rule – Lay buried, Lies Buried, etc.
The high costs to taxpayers were mostly a result of policies that guarantee cotton
farmers a portion of their projected revenue, rather than of coverage paying them
for damaged crops.

A rather than of coverage paying them for damaged crops


B instead of their coverage that pays for damaged crops
C as opposed to coverage paying them for damaged crops
D rather than coverage that pays them for damaged crops
E instead of coverage that pays them for damaged crop

Ans – D

Use of double of is wrong in A


portion of their projected revenue, rather than of coverage

Before looking at answer choices, you should take some time to unravel this tricky sentence.
Here is the core structure of the sentence: the costs were a result of polices that guarantee the
farmers THIS, rather than coverage that pays them THIS. It is easy to get lost in this sentence
and try to link improper structures. In (B) and (E) the “instead of” structure is problematic
because it illogically links it to the “of their projected revenue” portion before: “policies that
guarantee cotton farmers a portion of revenue, instead of coverage” is completely illogical and
does not contrast the policies with the coverage. (A) is more difficult to eliminate but lacks
parallelism with the first portion because of the improper “paying” in place of “that pays”. Also
the “of” after “rather than” is unnecessary (but could be used). In (C) “as opposed to” is an
awkward choice - “as opposed to coverage paying …” - and the “paying” is again incorrect. (D) is
correct as it conveys the meaning perfectly with parallel and consistent structures.

f you look closely, you will find that : mostly a result of (policies that guarantee cotton
farmers a portion of their projected revenue, rather than of coverage paying
them for damaged crops) .
Policies that guarantee is only parallel with coverage that pays . ( a and c is ruled out.)

"rather than" shows preference of one thing or the other whereas "instead of" suggests
replacement of a noun or an action noun with something else.

For example -
I’d call him a smart worker rather than a hard worker.
I prefer to have tea instead of coffee.

Also, please note that "instead of" is used to compare 2 nouns as of is a preposition. (refer to
the example above)

As a result of X, rather than Y..


The spokeswoman for the national laboratory announced that, while the lab had been operating
at a loss in the 1990s, it had become profitable in 2006, and would continue to yield high profits
at least until 2025.

A)had been operating at a loss in the 1990s, it had become profitable in 2006, and would
continue
B)had been operating at a loss in the 1990s, although becoming profitable in 2006, and will
continue
C)had been operating at a loss in the 1990s, it became profitable in 2006, continuing
D)operated at a loss in the 1990s, becoming profitable in 2006, then would continue
E)operated at a loss in the 1990s, becoming profitable in 2006, and continuing

Ans – A.

Reporting verb – announced and thus relevant tenses used.

The spokeswoman for the national laboratory announced that, while the lab had been operating
at a loss in the 1990s, it had become profitable in 2006, and would continue to yield high profits
at least until 2025.

Here the main sentence is


The spokeswoman for the national laboratory announced that it had become profitable in 2006,
and would continue to yield high profits at least until 2025.

- This is the core intention of author that he wanted to express

The part while the lab had been operating at a loss in the 1990s - is acting as a modifier here
which is giving some additional information about the earlier performance. So it is with
in , .... , structure.

Lucky's tutor recommends that, in order to score well on GMAT, Lucky should put in at least two
hours of practice everyday and not practice questions indiscriminately.

(A) Lucky should put in at least two hours of practice everyday and not practice questions
indiscriminately.

(B) Lucky should put in at least two hours of practice everyday and that he should not practice
questions indiscriminately.

(C) Lucky will have to put in at least two hours of practice everyday and not do practice
questions indiscriminately.

(D) Lucky put in at least two hours of practice everyday and that he not practice questions
indiscriminately.

(E) at least two hours of practice need to be put in by Lucky everyday and he does not have to
practice questions indiscriminately.
Ans – D

Command Subjunctive.

The point to note here is that the subjunctive mood has set in because it is the tutor who
recommends. A tutor’s recommendation is gospel. Consequently, we have to use the imperative
form of the verb that brings out a command. Therefore, D is good enough. On the contrary, B,
by using the word ‘should’, talks of recommendation as if it is a suggestion, which is not the
intent here. So, be is wrong.
We cannot say overall, a recommendation is not a command. It depends upon who makes that.
If it is a recommendation or advice by the RBI, it is a command to all banks. If the Supreme
Court recommends, it is a command to all lower courts. If the president advises it is a command.
B will not be subjunctive if it was Lucky’s friend who recommended that he should practice etc.
After all, Lucky is not bound to heed his friend’s advice.
As the prairies of the Midwest dried up because of drought and over-farming, many settlers of
the time where land was barren and homes had been seized in foreclosure moved further
westward in search of food and employment.

(A) where land was barren and homes

(B) where their land was barren and their homes

(C) with more barren land and homes that

(D) whose land was barren and whose homes

(E) having barren land and homes that

Ans – D

(C) with more barren land and homes that


("with" seems modifing "time" but it has to modify "settlers", even if "of the time"
cannot be placed anywhere else in the sentence without distoring the intended
meaning, better to use comma before "with" then it will clearly refer to setlers, but
this is not the solid point to eliminate this answer choice, the big pint is //, "homes
that had been seized in foreclosure" noun+modifier is not // to "more barren land"
adj+noun, hence (C) out)

My thoughts out loud:


"As" here presents two simultaneous actions in the past.
Usage of past perfect in non-underlined part seems not so critical and can be replaced with
simple past tense.
But I think the aouthor wanted to say that despite the homes of settlers had been seized in
foreclosure, settlers could live and do farming, but both "seized homes" and "barren land" was
tto much, and settlers decided to moved further westward in search of food and employment.
generis what do you think? Does this make sense?

As the prairies of the Midwest dried up because of drought and over-farming, many settlers of
the time where land was barren and homes had been seized in foreclosure moved further
westward in search of food and employment.

The sentence explain that some settlers moved further westward to escape the drought, the
seizures and the barren land. This already makes A and B senseless, because "where" can not
refer to people.
C is senseless also, if the have more barren land and homes that had been seized, why should
they move away? And also this does not express any kind of point in the sentence, but simply
modifies settlers describing their properties.
E si similar to C: it describes the properties of settlers but do not offer any kind of contrast. If E
was included between two points, it coulb be accetable, as it would become an indipendent
clause, but as it is, is not acceptable
In my opinion D is the correct one

(D) whose land was barren and whose homes


D explains that the lands of the settlers and their homes were unusable, and so they have been

The Kwakiutl recognized one social unit larger than the tribe-the confederacy, which was a
cluster of loosely knit, informally related neighboring tribes who interacted with themselves more
often than between other tribes.

(A) tribes who interacted with themselves more often than between
(B) tribes who interacted among each other more often than among
(C) tribes who interacted with one another more often than with
(D) tribes, interacting among each other more often than between
(E) tribes, interacting among one another more often than with

And – C

now, in a group people can interact "with one another", "amongst one another" whereas two
people interact "with each other" . Never can a group interact "with themselves". NEVER. so A,B
is out
C and D do not satisfy the grammar in the reduced sentence. they are clearly wrong.
only option C is left.
C is lucid and clear.

Here in the sentence we are talking about a cluster (more than 2 units/groups) , whenever we
compare more than 2 things we use 'one another' and when we compare between two
things/people/groups we use 'each other' .

To illustrate:X and Y were great friends and interacted with each other more than they
interacted with acquaintances (2 people)
X,Y,A,and B were great friends and interacted with one another more than they interacted
with acquaintances (more than 2 people)

And the idiomatic usage is 'interact with' not interact among. 

1. I was in a fix between options C and E but chose the incorrect answer because I
thought that "who" can't to tribes and we need which or that.( who can be only used
for people )
If the usage of 'who' is correct here, then 'who' should be able to refer to other
collective nouns such as a team, group etc?
generis

Well, until I saw this question, my answer would have been:


No, because who is used to refer only to nouns that are actually human beings.
That said, I can understand why GMAC used who in this case.

Which nouns are interacting? Individual members of the tribes.


The tribes themselves as a whole would not interact unless one whole tribe were negotiating a
treaty with another whole tribe, for example, or were arranging an alliance with another whole
tribe.
The people in the tribes are interacting.
I am okay with the use of who in this case and context.
Quote:
then 'who' should be able to refer to other collective nouns such as a team, group
etc

No, I would not go that far. One exception does not a make a rule.
If the context were similar to this one (individual members interact and it's kinda weird to think
of one whole group "interacting" with another whole group), then those other words you
listed might be able to take "who."

GMAT does not test the group THAT/who issue very often.
Furthermore, as far as I can recall, GMAT has not used who in this way in any other question.

Quote:
2. In option E, the verb-ing 'interacting' should make sense with
a. which was a cluster of loosely knit, informally related neighboring tribes-- verb-ing
'interacting' tries to refer to the subject of the previous clausewhich was..
b. or does the verb-ing modifier ' interacting' skips over [TO?] the preceding main
clause the Kwakiuti... ?

I think you are asking which clause the adverbial modifier "interacting among. . ." applies to.
I think you are also implying that "interacting" in (E) does not make a lot of sense. (If so, you
are correct.)

Yes, your (a) is correct. "Interacting" modifies the immediately preceding clause, and thus refers
to which  was.
Which, in turn, refers to confederacy.
And now the modifier makes no sense.
The confederacy . . ., interacting among one another . . .
The confederacy does not interact with or among anything or anyone.

Regarding your (b): If the participle "interacting" could reach "The Kwakiutl recognized . . . " —
doubtful, and the modifier interacting is still nonsensical.
Studies have found that deaf children from American families who identify themselves as
culturally deaf, and use sign language, are generally on a par with hearing children in terms of
reading and writing, because they have been exposed to language since birth.

A and use sign language, are generally


B using sign language, are generally
C use sign language, and are generally
D and using sign language are generally
E and use sign language, generally

Ans - A

I have few doubts in A :

1. children who identify themselves as culturally deaf, and use sign language --- why
comma is separating parallel verbs identify and use; sentence is not presenting list
of more than 3 items also not joining 2 ICs.

2. Why who is not repeated after and. as per MGMAT chapter 4 on parallelism


repetition of subordinatior (who/whose etc) is expected to clear any ambiguity. That
is why many people selected C.

children who identify themselves as culturally deaf and who use sign language are ...

I think both A and C have minor punctuation errors. Would someone like to clarify
this ?

Studies have found that deaf children from American families who identify
themselves as culturally deaf,  and use sign language, are generally  on a par with
hearing children in terms of reading and writing, because they have been exposed to
language since birth.

Hi PiyushK,
How r u 
Regarding point#2, hope below link form MGMAT answers it.I believe, there is an
excellent eGmat article also on this but somehow i cant find it right-now.
http://www.manhattangmat.com/forums/par ... 12348.html

Ralph likes people who are popular and who are tall.
HOWEVER, this is a different trap where there is ambiguity on whether he likes two different
types of people OR people who are two different things. To add clarity, we would distinguish as:
Ralph likes people who are popular and tall. (he likes people who are popular and tall)
Ralph likes people who are popular and people who are tall. (he likes people who are popular
and he also likes people who are tall)
Ralph likes people who are popular and who are tall. (ambiguity here... the GMAT might allow
this, but if given the preference chose a sentence with a clearer meaning)

As per this by dropping second who both identify and use get linked to the same subject deaf
children. Please share your views on this.

I think point number 1 is a very valid point. I did some research and figured out three things
1) Comma + and is used to join two independent clauses
2) Comma + and is used for closing a list of items
3) Since commas are used in pairs to enclose phrases that interrupt a clause , we can place a
comma before "and" (or any of the seven coordinating conjunctions) when the conjunction
launches such a phrase
example "I told him, and really meant this time, to not abusive language" . (Here two commas
are introducing non essential phrases)

Hi,
First of all,I have the same problem as PiyushK,but through the PEO I could find my choice.
The clue is in the non-underline part "because they have been exposed to language since
birth";This is the reason to what the studies have found.
If the intended meaning is to say that the studies have found deaf children doing 2 things:using
sign language,being on par,the reasoning cited in the latter part should response to this.
However,that reasoning part is actually explain only why these deaf children are on par with the
hearing children.Hence,there is only main verb in this sentence "are"
With this conclusion,option C,D,E are out.
Between A and B,option B has the //ism problem;hence only A survives. 
My guess on the comma before "and use sign" is to separate the non-essential information.Also
the ellipsis takes "who" from ",and use sign language".
Salsa Dancers warm up before every performance by doing a series of warmup and
stretching exercises, and it reduces the chance of injury.

(A) exercises, and it reduces


(B) exercises, which reduces
(C) exercises, reducing
(D) exercises, the routine reduces
(E) exercises, so the routine reduces

Ans – C

a) exercises, and it reduces -  "it" cannot refer to verb.


b) exercises, which reduces - "which" introduces a nonessential clause and modifies preceding
noun.
c) exercises, reducing - "reducing" modifies "warm up". nothing wrong with the option.
d) exercises, the routine reduces - I'm not sure that "routine" is unambiguous here. Comma
cannot connect two clauses. We need a semicolon instead of comma.
e) exercises, so the routine reduces - the same doubt about "routine".

C for me
A - pronoun error (it)
D - run-on sentence - if it were "the routine reducing", D would be an attractive choice
E - ..., so ... distort the meaning by reversing cause and effect.
because of reducing injury, they do warm up

B and C are close.


But I choose C because "Salsa Dancers warm up before every performance" is the true factor
that reduces that injury
In B, how can just a series of warmup reduce an injury? It should contain a word that
demonstrates "routine," such as everyday

The series of warmup and stretching exercises--> will reduce the chance of injury. This
prevention of injury is the outcome of exercises.

In Option D ------> The routine reduces makes a Independent clause which is not connected to
first IC correctly.

A semicolon would be a much better option. Another way, the Option D could have been written
"the routine that reduces...." which means noun+noun modifier construction modifying the noun
" warm and stretching exercises"

(B) exercises, which reduces

which here refers to 'exercises'


1. if we see grammatically then plural noun + singular verb(reduces) wrong.
2. If we understand logically then 'warm up' before performance reduces the chance of injury
not exercises.
Salsa Dancers warm up before every performance by doing a series of warmup and stretching
exercises
and
the routine reduces the chance of injury

are two independent clauses so you need a coordinator, a subordinator or a semicolon to join
them

A recent article in The New York Times reported that many recent college graduates had
decided on moving back into their parents’ home rather than face the
uncertainty and expense of the rental market.

A) had decided on moving back into their parents’ home rather than face
B) had decided on moving back into their parents’ homes instead of facing
C) have decided to move back into their parents’ homes instead of facing
D) had decided to move back into their parents’ homes rather than facing
E) have decided to move back into their parents’ homes rather than face

Ans - E

Split 1

Had vs Have
Past perfect does not make sense here.

Had decided  means the action of deciding happened before. Though, the sentence does not
clarify before what. Did it happen before the article reported. Does not make sense to say that.

So we can eliminate A, B, D

Split 2

instead of vs rather than

instead of needs to have a noun - not the case here


rather than is flexible and can be followed by noun/clause/verb

So correct choice is E

It's true that technically, we can use either present or past perfect (have or had) when we
describe one past action that precedes another. However, it's important to understand that we
don't usually need to use past perfect. If we have more than one past action, we can use past
perfect to indicate that one action preceded the other. However, just because we can use past
perfect doesn't mean we should. Typically, we will only do this if a) the sequence matters and b)
the prior action is over. This sentence meets neither of those criteria. If we read an article about
something, clearly that thing occurred before the article was written, so there's no need to clarify
the sequence. Additionally, since this article describes a current trend, presumably college
graduates continue to make this decision. (If the article described a decision that were made at
one point in time, such as the decision of a jury, past perfect would make more sense.) This
renders the past perfect completely inappropriate here. The use of "had" is incorrect in this
context.

It's worth noting that this is a poor and rather transparent rewrite of an official GMAT
question: https://gmatclub.com/forum/a-recent-stu ... 73334.html

Notice that the original makes things clearer with a modifier ("within the past few years") that
makes the use of the past perfect absolutely impossible. It also opens with the present perfect,
although it's possible for a recent study to "have found" something out about the past, the
present, or even the future. In other words, opening with the present perfect doesn't force the
other verbs to be in the present perfect.

By the way, pulkitaggi, you're right that we don't need to use tense to get to E. Only E has
the parallel form "to move . . . rather than face."

And similar Q is as below:

A recent study has found that within the past few years, many doctors had elected early
retirement rather than face the threats of lawsuits and the rising costs of malpractice insurance.

(A) had elected early retirement rather than face


(B) had elected early retirement instead of facing
(C) have elected retiring early instead of facing
(D) have elected to retire early rather than facing
(E) have elected to retire early rather than face

Ans - E

A March 2000 Census Bureau survey showed that Mexico accounted for more than a quarter of
all foreign-born residents of the United States, the largest share for any country to
contribute since 1890, when about 30 percent of the country’s foreign-born population was from
Germany.

(A) the largest share for any country to contribute

(B) the largest share that any country has contributed

(C) which makes it the largest share for any country to contribute

(D) having the largest share to be contributed by any country

(E) having the largest share to have been contributed by any country
Ans - B
The OE for this is that both HAD or HAS could have been right in option B, depending on when
the Question was made and on whether the fact is true today also or not.

We now have wireless computer networks, direct-broadcast satellite television, digital wireless
cable television networks, global telephone service, and global positioning systems that can
pinpoint an individual’s location to a few inches.
A. that can pinpoint an individual’s location to a few inches
B. that can pinpoint an individual’s location in a few inches
C. which can pinpoint an individual’s location to a few inches
D. which can pinpoint an individual’s location of a few inches
E. which should pinpoint an individual’s location to a few inches

THAT Vs WHICH
THAT without comma should be used as essential modifier.
WHICH with commas should be used as non-essential modifier.

Here we can clearly see that modifier is essential to complete he sentence, also comma is not
provided before which. So we have to use THAT

We know; "that without comma" or "comma+which" create relative sentences that refer to noun
or noun-phrase immediately preceding it.

thus, C.D.E. are all out for there is no 'comma' before which. We are left with A & B, wherein
'that' clause modifies "global positioning systems'. . next, "pinpoint ...to" is idiomatic.

Here we can easily eliminate those choices that have which since you would need a comma
before which to make the sentence correct. Besides, the rest of the sentence is essential to the
other part of the sentence so that must be used. C, D and E are out for these reasons.

In B, "that can pinpoint an individual’s location in a few inches", in is not the right word to use
here. You can use in when you're talking about time  not locations and therefore would make
A the right choice given that to is properly used to express distance or location.
The Near Earth Asteroid Rendezvous (NEAR) spacecraft will orbit the asteroid Eros for a year,
slowly moving closer to the surface of the object to make ever more precise measurements that
scientists hope will enable them to understand how the solar system formed some four billion
years ago.

A. to make ever more precise measurements that scientists hope will enable them to

B. to make ever more and more precise measurements, which scientists are hoping to enable
them

C. for making ever more precise measurements, and scientists hope that they will be able to

D. with the purpose of making more precise measurements than ever, and which scientists hope
will enable them to

E. in order to make more precise measurements than it ever did, and scientists are hoping they
will be able to
Ans – A

A. to make ever more precise measurements that scientists hope will enable them to
"ever more..." is an acceptable idiomatic expression. its meaning is roughly the same as that of
"increasingly" or "more and more".
* "them" refers to "scientists". (if you actually omitted "scientists hope", then you would have to
replace "them" by "scientists".)

B. to make ever more and more precise measurements, which scientists are hoping to enable
them
* REDUNDANCY: you can't combine 2 idioms that mean the same thing.
you say EITHER "ever more" OR "more and more", but not both.
* "measurements, which scientists are hoping to enable them" doesn't make sense

C. for making ever more precise measurements, and scientists hope that they will be able to
* "for making" is not idiomatic. this should be the infinitive "to make".
* the "and" connector is inappropriate here. if you use "and" like this, then you're implying that
there are two ENTIRELY SEPARATE things going on: (a) the spacecraft is hanging around
making measurements, and (b) scientists hope one day that blah blah blah.
these are clearly connected, so, "and" is inappropriate.

D. with the purpose of making more precise measurements than ever, and which scientists hope
will enable them to
* not parallel.
you can't place a prepositional phrase (with the purpose of...) in parallel with a relative-pronoun
clause (which ...).

E. in order to make more precise measurements than it ever did, and scientists are hoping they
will be able to
same problem with "and" as in (c).
also unacceptably wordy / awkward.

Like Haydn, Schubert wrote a great deal for the stage, but he is remembered principally for his
chamber and concert-hall music.

(A) Like Haydn, Schubert


(B) Like Haydn, Schubert also
(C) As has Haydn, Schubert
(D) As did Haydn, Schubert also
(E) As Haydn did, Schubert also

Ans – A

Try questioning yourself--is also really required? Like Haydn, Schubert did something does it all!
Like Haydn, Schubert also did something... is also required here? Like by itself does the job.

B - Redundancy
C - wrong tense, 'as' compares actions (verbs)
D - redundancy and action oriented
E - redundancy and action oriented

You should not START a sentence with "as + HELPING VERB + subject", or with "as + subject +
HELPING VERB". in those instances, the sentence is better written with "like + SUBJECT".

Example:
as did his brother, james graduated at the head of his class --> wordy/awkward/inferior
like his brother, james graduated at the head of his class --> better.

tructurally, we can use “as” as well to present the comparison. But in that case “as” should be
followed by a clause.

LIKE is used:
To compare 2 nouns
In Hypothetical situations
When there is no verb following LIKE.
metamorphically comparisons

AS is used:
To compare actions / verbs / clauses / trends / prepositional phrases
To denote real situations
To denote functions/professions/ designations / jobs / roles etc
A wave of new seltzer bars and bottle shops — all opened in the last two years, with some
serving mostly alcoholic versions while others do not — are fueling interest in this formerly low-
prestige beverage.

(A) with some serving mostly alcoholic versions while others do not — are

(B) with some serving mostly alcoholic versions and others not — are

(C) and some serving mostly alcoholic versions while others do not — is

(D) some serving mostly alcoholic versions and others not — is

(E) with some serving mostly alcoholic versions and others do not — is

The choice between (C), (D), and (E) is more difficult. While there are several differences
between the choices, the one that matters is the choice between “while/and others do not” and
“others not”. In a case like this, when you are omitting the verb form from the second part, you
need to make sure the form you are using matches the form used previously. An easy way to
check that is to simply put the original form where it is omitted and see if it works.

In this sentence, you have two choices:

“some serving mostly alcoholic versions while/and others do not SERVING them” NO – this does
not work

OR

“some serving mostly alcoholic versions and others not SERVING them” YES – this works.

As a result, the correct answer is (D) as it is the only one with proper subject-verb agreement
and a parallel construction where the verb form is omitted.

The two parts have to be parallel when it comes to verb as there is no need to change the
verb tense. If the tense need to be changed, verb form will have to be repeated.
Whereas high speeds generally cannot be achieved by ramjets without their initially being
assisted by a rocket, a scramjet, or supersonic combustion ramjet, can attain high speeds by
reducing airflow compression at the entrance of the engine and letting air pass through at
supersonic speeds.

(A) Whereas high speeds generally cannot be achieved by ramjets without their initially being
assisted by

(B) Whereas a ramjet generally cannot achieve high speeds without the initial assistance of

(C) With ramjets, high speeds generally are not able to be achieved without initially being
assisted by

(D) Unlike the ramjets, generally unable to achieve high speeds without the initial assistance of

(E) Although a ramjet generally does not have the capability of achieving high speeds without
the initial assistance from

Ans – B

A few issues with D:

*"Unlike" is a problem because it makes us compare ramjets to ramjets! Notice that a scramjet is
a kind of ramjet. It would be like saying "Unlike dogs, chihuahuas, which are small dogs, do not
need much food." Chihuahuas may be unlike *most* dogs, but we can't say they are unlike dogs
in general!

*"The ramjets" is wrong. We aren't talking about a certain group of ramjets, so "the" is
inappropriate. It's like saying "I like the apples." If you aren't talking about a specific group of
apples, it makes no sense.

*The modifier starting with "generally unable" is wrong. It is taking the form of an adverbial
modifier, so we'd actually expect it to modify the following clause: "a scramjet . . . can attain
high speeds." So we're saying "Unable to achieve high speeds, a scramjet can attain high
speeds." Of course this is impossible. Now if what I'm saying doesn't seem accurate to you, it's
perhaps because we have *two* initial modifiers in a row in this choice, making it hard to tell
what the sentence is really trying to say.

It's fine to compare a plural noun with a singular noun,


Not only speech enables man to handle and pass on more complicated ideas, it also enables him
to accumulate them.

a) Not only speech enables man to handle and pass on


b) Not only does speech enable man to handle and pass on
c) Not only is speech enabling man to handle and pass on
d) Not only speech enables man to handle and pass off
e) Not only speech enables a man to handle and pass on

Ans – B

Generally:
When a sentence begins with a NEGATIVE -- not, never, seldom -- a helping verb will PRECEDE
the subject.
Never HAS John won two rounds.
Not only HAS Mary been promoted to vice president, she has also received a raise.
Seldom DID the team qualify for the playoffs.

The first clause conveys the following meaning:


Speech  does enable  a man to handle and pass on more complicated ideas.
The entire portion in blue -- does enable -- serves as a verb for speech.

Main point in this Sentence is to notice that we need to compare Verb and not noun . i.e What
all speech can enable. It enables man to handle and pass on .. and it also enables him to
accumulate them .
However, the problem with the main sentence is that it is trying to compare speech with enable ,
which is weird .
When the reporter asked about the status of the budget, the governor said that, at that very
moment, his team is fashioning a compromise and will deliver it to the senate offices by later
that afternoon.

(A) is fashioning a compromise and will deliver


(B) fashions a compromise and will deliver
(C) was fashioning a compromise and would deliver
(D) had been fashioning a compromise and would deliver
(E) has fashioned a compromise and will deliver

Ans - C

Importance of – at that very moment:


yeah it is important. in fact without this phrase i wouldn't have been able to decide
between "was fashioning" AND "had been fashioning" . the reasoning is as follows:
as i have said in reported speech the tenses makes a usual jump. now that is a very crude way
of saying but what does that actually mean?
suppose i met the governor and he said following thing to me:   "at that very moment, his team is
fashioning...."  then that would become in reported speech as  "at that very moment, his team
was fashioning...."-----> now the important role that the phrase "at that very moment" is playing
is that it is guiding me to the fact that the manager was talking in present progressive tense.
without this phrase i am left to a possibility that he might be talking in "past progressive", in that
case my reported speech would have been "had been fashioning ......"
NOW the important point is has GMAC ever tested such constructions? i do not remember any
official problem that has tested such a nuance.
i will appreciate if the makers of this question (MIKE & TEAM) can chime in to further elaborate.

In spite of the fact that Justin had sprained his ankle early in the baseball game, he continued
playing and even scored the winning run.
a) In spite of the fact that Justin had sprained
b) Because of the fact that Justin sprained
c) Despite Justin had sprained
d) Although the spraining that Justin has done to
e) Even though the fact was that Justin was sprai

Ans- A

Despite is a preposition. As per thumb rule, all prepositions have to be followed by nouns or
noun phrases. Justin had sprained is a clause that can not legally follow despite. So, C is wrong

"Despite" can be used here, but not in that particular phrase. "Despite Justin having sprained his
ankle early..." would work; "Despite Justin had sprained" does not.
Both despite and in spite of are prepositions; both are inter- changeable. It may be seen that
choice A talks about in spite of ‘the fact’; the fact is the noun that follows theprepositional
phrase ’in spite of’; the relative clause that follows is a modifier of the noun ‘the fact’ . Hence, it
is perfectly acceptable.

Once each vote and piece of paper are burned, the cardinals in the conclave are given time to
reflect before casting their next vote.

A) Once each vote and piece of paper are burned

B) Once each vote and piece of paper is burned

C) Once each vote and piece of paper has been burning

D) Once each vote and pieces of paper are burned

E) Once each votes and pieces of paper are burned

Ans – B

Each DOES refer to piece of paper.


Each carries over to or "distributes" to both subjects.

I am going to use some jargon.


In this case, jargon should make the issue easier to understand.

The word each can be a determiner.


A determiner identifies a noun: This question (not that question). Some people (not all people).

Each and every are called "universal" determiners; both words "grab" all the members of a
group.
Often they are interchangeable, though they have slightly different emphases and each can refer
to two things, whereas every must refer to three or more things.
-- Each artist interprets the world differently.  (Every single artist has a different interpretation of
the world.)
-- Every artist is sensitive. (All artists possess a sensitive temperament.)

When each and every are placed before the subject (are "preposed," which means "positioned


before"), both words have a "distributive effect" and require a singular verb.

(1) Each distributes to both nouns in a compound subject


Each placed before vote and piece of paper "distributes" to both nouns and means
Each vote and each piece of paper.

(2) As a determiner, when each is placed before either a single subject or a


compound subject, each takes a singular verb.

If we write each before both parts of the compound subject, the verb is still singular.
Correct: Once each vote and each piece of paper IS burned ...
If we write each before only the first noun in the compound subject, the verb is still singular.
Correct: Once each vote and piece of paper IS burned ...

Each or every positioned before a list of nouns (1) distributes (applies) to all the nouns and (2)
takes a singular verb.**

This way is correct (I will emphasize that each controls the verb):


Once EACH [vote and piece of paper] IS burned, the cardinals . . .

Linking arrangements among secondary schools and the workplace never evolved in the United
States as they have in most other developed countries.

(A) among secondary schools and the workplace never evolved in the United States as they have
(B) in the United States among secondary schools and the workplace never evolved as they did
(C) between secondary schools and the workplace never evolved in the United States as
(D) in the United States between secondary schools and the workplace never evolved as they
have
(E) between secondary schools and the workplace never evolved in the United States as they did

Ans – E

One might get the feeling that C and E mean the same thing. It is not so in that C has a subtle
but substantial error of intent. C says that the listing arrangements never evolved in the US just
as in the same way the arrangements did not evolve in most other developed countries. In other
words, it tends to mean that in other countries also they did not evolve. This is a grave flaw. The
original intent is that while in other countries the arrangements evolved smoothly, in the US,
they did not.

The largest trade-book publisher in the United States has announced the creation of a new
digital imprint division, under which it will publish about 20 purely digital works to be sold online
as either electronic books or downloadable copies that can be printed upon purchase.

(A) works to be sold online as either electronic books or


(B) works to sell them online, either as electronic books or
(C) works and it will sell them online as either electronic books or as
(D) works, and selling them online as either electronic books or as
(E) works, and it will sell them online either as electronic books or

Ans - A

he important thing to note in this context is that by using the passive voice ‘to be sold’ the text
does not say who is going to sell the digital works. To assume that the company itself is going to
sell them is a significant change of meaning. That is the reason choices B, C, D, and E fall by the
wayside. D in addition is a fragment. Then add to this the additional grammar requirement of
correlative conjunction parallelism and you will find A glaring as the right choice, even dispelling
the close E.

The catch here is the while the original simply says that the company will publish the digital
works to be sold, it does not state that the company itself is going to sell them. May be the
selling is going to be through some third party arrangement, whereas, E categorically states that
the company will sell them; this is a deliberate distortion of the original intent and hence E is not
to be preferred

Yes. You are absolutely right; both are correct as per the tenets of correlative conjunctional use.
What comes after Either should be repeated after Or

While Jackie Robinson was a Brooklyn Dodger, his courage in the face of physical threats and
verbal attacks was not unlike that of Rosa Parks, who refused to move to the back of a bus in
Montgomery, Alabama.

(A) not unlike that of Rosa Parks, who refused


(B) not unlike Rosa Parks, who refused
(C) like Rosa Parks and her refusal
(D) like that of Rosa Parks for refusing
(E) as that of Rosa Parks, who refused

Abs – A

Though unusual for using a double negative such as 'not unlike' to denote a positive factor “like”,
A is the best of the choices as all others fault on various counts;
B and C are comparing courage to Rosa Parks
D, uses a gerund ‘for refusing’ and muddles up the meaning by not making clear who exactly did
not move to the back of the bus, whether Jackie or Rosa
E uses as for comparing nouns

A 2013 agreement among North American governments reduced the quantity of tomatoes that
Mexican producers had been allowed to sell at below-market prices in the United States.

(A) reduced the quantity of tomatoes that Mexican producers had been allowed to sell
(B) reduces the quantity of tomatoes allowed for sale by Mexican producers
(C) reduced the tomato quantities that Mexican producers had been selling
(D) reduced the quantity of tomatoes that Mexican producers are allowed to sell
(E) reduces the tomato quantities Mexican producers have been allowed to sell

Ans – D

(A) reduced the quantity of tomatoes that Mexican producers had been allowed to sell - we
need to be careful here. This past perfect makes sentence look meaningless because
if the agreement took place later than sellers had already been selling some quantity,
then how the agreement can reduce the quantity that was ALREADY sold. Thus, out
(B) reduces the quantity of tomatoes allowed for sale by Mexican producers - 'reduces' is
wrong because the agreement took place back in 2013, thus we need past tense and
not simple present. Out
(C) reduced the tomato quantities that Mexican producers had been selling - Same as A for
past perfect. Out
(D) reduced the quantity of tomatoes that Mexican producers are allowed to sell - both tenses
are correct: one is 'reduced' because agreement took place in 2013 and 'they are still
allowed to sell, thus present continuous. Correct
(E) reduces the tomato quantities Mexican producers have been allowed to sell - 'reduces' is
wrong as described in B

Archaeologist Donald Johanson wrote of his team's discovery of a 3.2 million-year-old skeleton in
his award-winning book Lucy: The Beginnings of Humankind.

(A) Archaeologist Donald Johanson wrote of his team's discovery of a 3.2 million-year-old
skeleton in his award-winning book Lucy: The Beginnings of Humankind.

(B) In his award-winning book Lucy: The Beginnings of Humankind, archaeologist Donald


Johanson wrote of the discovery by his team of a 3.2 million-year-old skeleton.

(C) Writing the award-winning book Lucy: The Beginnings of Humankind, archaeologist Donald
Johanson chronicled the discovery of a 3.2 million-year-old skeleton by his team.

(D) The discovery of a 3.2 million-year-old skeleton was chronicled in the award-winning
book Lucy: The Beginnings of Humankind, being written by the archaeologist Donald Johanson.

(E) In his award-winning book Lucy: The Beginnings of Humankind, archaeologist Donald


Johanson wrote of his team's discovery of a 3.2 million-year-old skeleton.

Ans – E

A prepositional phrase may be used as a noun modifier or a verb modifier. As a noun modifier, it
refers to the noun it touches, and as a verb modifier, it refers to the verb of the associated
clause (placement of a verb modifier is flexible - it may be placed at the beginning, at the end or
in the middle of the clause). In order to avoid ambiguity, when such a prepositional phrase is
used a verb modifier, it is preferable to shift the modifier away from any noun that it may refer
to.

In option A the prepositional phrase modifier "in his award-winning book Lucy: The Beginnings
of Humankind" is used as verb modifier to refer to the verb "wrote". So it is preferable to shift it
away from the noun "skeleton". Otherwise an ambiguity may arise whether it refers to the verb "
wrote" or to the noun " skeleton" (the latter wrongly implying that the skeleton is in the book)

A.Archaeologist Donald Johanson wrote of his team's discovery of a 3.2 million-year-old


skeleton in his award-winning book Lucy: The Beginnings of Humankind.
This choice illogically suggests that the discovery took place in the book.

B.In his award-winning book Lucy: The Beginnings of Humankind, archaeologist Donald
Johanson wrote of the discovery by his team of a 3.2 million-year-old skeleton.
Team of 3.2 million skeleton. Try to place prep. phrases close to the known it modifies.

C.Writing the award-winning book Lucy: The Beginnings of Humankind, archaeologist Donald


Johanson chronicled the discovery of a 3.2 million-year-old skeleton by his team.
writings and chronicle provide the same meaning and hence provide a case of redundancy.

D.The discovery of a 3.2 million-year-old skeleton was chronicled in the award-winning book
Lucy: The Beginnings of Humankind, being written by the archaeologist Donald Johanson.
The same error is present here also. Moreover, this choice also uses passive, which generally
should be avoided.

The parachutelike membranes of Africa's scaly-tailed flying squirrels differ from those of the
flying squirrels in the family Sciuridae because they are attached to a cartilage rod at the elbow
instead of the wrist.

(A) because they are attached to a cartilage rod at the elbow instead of

(B) because of being attached to a cartilage rod at the elbow rather than at

(C) in their attachment to a cartilage rod at the elbow instead of being attached at

(D) in that they are attached to a cartilage rod at the elbow rather than at

(E) in their being attached to a cartilage rod at the elbow instead of

Ans – D

Option A is wrong because of the use of "because" .....The membranes differ not because they
are attached to cartilage, but because of some evolutionary and adaptive factors. The required
phrase should answer the questions in what way they differ, not why they differ. Therefore the
use of " in that" is better than use of "because".

One may replace "instead of" with "rather than" in all cases (although the vice versa is not true).

"Instead of" is applicable for nouns only. "Rather than" is applicable for nouns and all others
(including prepositional phrases as in this case). Hence use of "instead of" would be wrong in
option D (used for prepositional phrase at the wrist).

Of all the people with whom you and I come in contact in our dentist practice, it is the youngest
patients who often have the most contagious illnesses, so be sure to wear your latex gloves and
dental face mask when working on pediatric patients

A) with whom you and I come in contact

B) with who you and I come in contact

C) with whom me and you come in contact

D) with who me and you come in contact

E) who you and I come in contact with

And – A
• Split #1 - the preposition WITH requires an OBJECT PRONOUN

Of all the people with ___  [whom or who?]

WITH is a preposition.
Prepositions require objects.
With requires the object pronoun whom.

Subject pronouns: I, you, she, he, it, we, they, who

Object pronouns: Me, you, her, him, it, us, them, whom

No matter where the preposition with is placed in a sentence, it requires an object:


with SOMEONE or SOMETHING.

with whom is correct.


Whom refers to the people just mentioned.

Options B, D, and E incorrectly use WHO.


Eliminate them

• Split #2 - only SUBJECTS can take a verb such as "come in contact with"

Subjects can take a verb. Objects and object pronouns cannot.


ME does not do anything. Ever.

We need subject pronouns to do the action of the verb.


Two dentists come in contact with others.
Those two dentists are subjects. Subjects do actions.

(Objects receive action.)

Use the subject pronouns you and I.

Option C incorrectly uses me (as does D, which is already gone).


Eliminate C.

New fall shows added to the television network’s upcoming weeknight schedule have increased
overall viewership, critical acclaim, popularity among all age groups, and have decreased ratings
on the network’s shows online

A) popularity among all age groups, and have decreased

B) popularity among all age groups, and decreased

C) and popularity among all age groups, while decreasing

D) increased popularity among all age groups, but decreased

E) and popularity among all age groups were increased, while decreasing

Ans – C
New fall shows
Have increased A, B, and C
And Have decreased D

A) popularity among all age groups, and have decreased - And is missing before third item in
sub-list.
B) popularity among all age groups, and decreased -Same as A
C) and popularity among all age groups, while decreasing - Correct.
D) increased popularity among all age groups, but decreased - Shows have increased ''Increased
popularity''...seems wrong and redundant. And is required to separate items.

Similar to rising interest rates, consumer and producer prices have been rising.

(A) Similar to rising interest rates, consumer and producer prices have been rising.

(B) Consumer and producer prices have been rising, as have interest rates.

(C) As interest rates are rising, so have consumer and producer prices.

(D) Consumer and producer prices have been rising, like interest rates do.

(E) Consumer and producer prices, as interest rates, have been rising.

Ans – B

A) Similar to means same as Like


B) Looks fine to me
C) I believe is not a complete sentence since neither clause is an independent clause. I also
believe that the are/have verb tenses conflict. [someone please confirm]
D) Should be AS instead of LIKE
E) Should be LIKE instead of AS

The French general Henary Petain, a highly-decorated world war I military leader, had his
reputation ruined because of his collaboration with Nazis during the german occupation in World
War II.

A) The French general Henary Petain, a highly-decorated world war I military leader
B) Hanary Patin, a French general who was a highly-decorated world war I military leader
C) Highly-decorated world war I military leader, the French general Hanary Patin,
D) The French general Henary Petain who was a highly-decorated world war I military leader
E) The highly-decorated French general Henary Petain, a military leader for world war I

Ans – A

A. The French general Henry Petain, a highly-decorated World War I military leader: Correct for
reasons stated above.

B. Henry Petain, a French general who was a highly-decorated World War I military
leader: Incorrect. The relative pronoun clause “who… leader” makes this information non-
essential for the sentence. The sentence now seems to say that Henry Petain was a French
general who had his reputation ruined for this reason. By the way, he was a highly decorated
World War I military leader. This is not the intended meaning. Petain’s being a highly decorated
WW I leader sets the contrast. Once a celebrated leader, Petain lost his reputation later.

C. Highly-decorated World War I military leader, the French general Henry


Petain: Incorrect. Remember we have still have the comma before the verb “had”. In this choice,
the subject and the verb are separated with a comma. This is not a grammatical construction.
The opening verb-ed modifier is correctly modifying the subject.

D. The French general Henry Petain, who was a highly-decorated military leader for World War
I: Incorrect.
1. Repeats the same “who” error of choice B.
2. Use of preposition “for” is incorrect.

E. The highly-decorated French general Henri Petain, a military leader for World War
I: Incorrect.
1. Modifier “the highly decorated” is now placed before “the French general Henri Petain”. The
choice now says that Retain was a highly decorated French general and was just a leader in
World War I. It does not say that he was a highly-decorated leader in WW I. This again reduces
the contrast that he lost his reputation in WW II.
2. Repeats the preposition error of Choice D.

“had” in this sentence is not a past perfect tense. It is a simple past tense verb for “have/has”.
“Has/have/had” only become past perfect tense when they are followed by another verb.

Notice that “ruined” here is a verb-ed modifier that modifies the preceding noun “reputation”.

Also, it does not make sense for the sentence to use past perfect tense because this sentence is
giving us general information about Henry Petain. Hence, the sentence is correctly written in
simple past tense.

Again, there is just one verb in this sentence. Past perfect is used to establish time sequencing
between two past events.

During the 1980’s approximately $50 billion in private investment capital is estimated to have left
Mexico and added to the strain on the country’s debt-ridden economy.

(A) During the 1980’s approximately $50 billion in private investment capital is estimated to have
left Mexico and added

(B) During the 1980’s it is estimated that approximately $50 billion in private investment capital
left Mexico and added

(C) It is estimated that there was approximately $50 billion in private investment capital that left
Mexico during the 1980’s and added

(D) It is estimated that during the 1980’s approximately $50 billion in private investment capital
left Mexico, adding

(E) Approximately $50 billion in private investment capital is estimated as having left Mexico
during the 1980’s, adding

Ans – D
During the 1980’s approximately $50 billion in private investment capital is estimated to have left
Mexico and added to the strain on the country’s debt-ridden economy.

The two major issues here are Modifiers and Parallelism. If you don't see that...it's time to buy
the MGMAT SC guide! : )

(A) During the 1980’s approximately $50 billion in private investment capital is estimated to have
left Mexico and added
PROBLEMS:

MODIFIER: Prepositional phrases will often be bounced around sentences. This will necessarily
change the meaning. DO NOT think that it doesn't matter where these prepositional phrases go.
The phrase here is "During the 1980s" which is modifying the verb "is estimated" in this answer
choice. But the estimation didn't happen during the 1890s, the money left during the 1980s.
ALSO, the word "have" is not incorrect. It is neither singular nor plural, but part of the infinitive
form "to have".

PARALLEL: The capital did not "leave Mexico" and "add strain". The FACT that money
disappeared added strain, not the money itself.

(B) During the 1980’s it is estimated that approximately $50 billion in private investment capital
left Mexico and added

PROBLEMS:

MODIFIER: Same as A.

PARALLEL: Same as A.

(C) It is estimated that there was approximately $50 billion in private investment capital that left
Mexico during the 1980’s and added

PROBLEMS:

MODIFIER: This issue is fixed because the "during the 1980's" comes AFTER the estimation now.

PARALLEL: Same as A.

(D) It is estimated that during the 1980’s approximately $50 billion in private investment capital
left Mexico, adding

CORRECT ANSWER!

MODIFIER: This issue is fixed, because the "during the 1980s" comes AFTER the word estimated
now.

PARALLEL: Instead of keeping capital as the subject of the verb "add", we've changed the verb
"add" to the participle/modifier "adding", which modifies the FACT that the money left Mexico.
This is what we were looking for all along.

(E) Approximately $50 billion in private investment capital is estimated as having left Mexico
during the 1980’s, adding

MODIFIER: During the 198s is okay.


PARALLEL: "adding" is oka.

IDIOM: You have to say "estimate to", not "estimate as" or even "estimate at".

Q1. Using a seismic survey, hydrocarbons can be located even though they are buried far
beneath Earth's surface.

a) Using a seismic survey, hydrocarbons can be located even though they are buried far beneath
Earth's surface
b) Hydrocarbons can be located even though they are buried far beneath Earth's surface, using a
seismic survey
c) Locating hydrocarbons that are buried far beneath Earth's surface, a geophysicist can use a
seismic survey
d) Buried far beneath Earth's surface, hydrocarbons can be detected using a seismic survey
e) Using a seismic survey, a geophysicist can locate hydrocarbons even though they are buried
far beneath Earth's surface

Ans: Clearly the statement is missing the subject that the modifier "Using a seismic survey" is
modifying. So a) is incorrect. e) is the best option here.
However, in passive forms of sentences, the subject is optional. So the below two forms, if
given, would also have been correct:

1. Hydrocarbons can be located using a seismic survey even though they are buried far beneath
Earth's surface (re-shuffled option b)
2. Hydrocarbons, buried far beneath Earth's surface, can be detected using a seismic survey (re-
shuffled option d)

Proximity of Noun Modifiers to the Nouns they modify


Simple Example

Let’s bring back the simple example that we discussed earlier. In this example, it makes sense
for the “noun modifier” to modify a slightly far-away noun.

1. The committee chose Mr. Smith of Left Block, who was the most experienced member, to lead
all the management-related operations.

We have already seen how “who” correctly modifies “Mr. Smith” in this sentence. Compare this
with the following sentence:

2. The committee chose Mr. Smith in the last meeting, who was the most experienced member,
to lead all the management-related operations.

If you notice, structurally there is no difference between sentences 1 and 2. In both, “Mr. Smith”
is followed by prepositional phrase. However, the second sentence is not correct. Here “who”
ends up modifying immediately preceding noun “the last meeting”, resulting in modifier error.
This is so because “in the last meeting” does not modify “Mr. Smith”. It rather modifies the
action “chose”. When did the committee choose? It did in the last meeting. This prepositional
phrase can actually be placed right in the beginning of the sentence, after “The committee”, or
before “Mr. Smith” to convey the intended meaning. Hence, here “who” cannot jump over the
preceding noun. Notice how per the context of this sentence, the expression “Mr. Smith in the
last meeting” is not a noun phrase. Contrast this with the noun phrase in the original sentence
“Mr. Smith of Left Block”.

1. The decision of the European leaders to use the Continent’s bailout funds to recapitalize
struggling banks would provide help to banks without directly adding to the sovereign debt of
countries, which has been a problem for Spain and potentially for Italy.

I believe the above sentence is right as which in the sentence refers to the Sovereign debt. The
Propositional phrase "Of Countries" modifies Sovereign Dept hence can not be placed anywhere
else.

2: The decision of the European leaders to use the Continent’s bailout funds to recapitalize
struggling banks would provide help to banks without directly adding to the sovereign debt in
the coming months, which has been a problem for Spain and potentially for Italy.

In the above sentence, which can not refer to sovereign debt because propositional phrase "in
the coming month" is referring to "would provide help to banks". Hence, "which" can not refer t
the Sovereign Debt.

Visitors to the park have often looked up into the leafy canopy and saw monkeys
sleeping on the branches, whose arms and legs hang like socks on a clothesline.

(A) saw monkeys sleeping on the branches, whose arms and legs hang
(B) saw monkeys sleeping on the branches, whose arms and legs were hanging
(C) saw monkeys sleeping on the branches, with arms and legs hanging
(D) seen monkeys sleeping on the branches, with arms and legs hanging
(E) seen monkeys sleeping on the branches, whose arms and legs have hung

Ans - D

I have two doubts in this question:


1) According to the OE, "whose" modifies "branches". However, in other official
questions I have seen that sometimes the clause modifier doesn't modify the closest
noun. Actually, it can modify the main noun in a noun phrase as long as it makes
sense and it is not ambiguous.
In this case, we have "monkeys sleeping on the branches, whose..."; "monkeys" is
the main noun of that noun phrase and makes sense with "arms and legs hang like
socks on a clothesline".
So, my question is: Is the split "whose....", a good reason to eliminate some choices?

2) What are the rules related to a prepositional phrase after a comma? In this
question: "..., with arms and legs hanging". How could we know that the
prepositional phrase refers to the monkeys and not the visitors. Because these
modifiers modify the entire clause, usually modifies the subject (visitors).
On the GMAT I think whose can only refer to people, and not animals. Saw can be eliminated
because it causes a conflict in tense. Basically if the visitors saw a monkey the next set of
customers may not see the same monkeys hanging on trees. And the action is ongoing because
the continually have people coming and noticing the same as the last group.

So A, B, and E can be eliminated because whose is not referring to a person. A, B and C can be
eliminated because it
changes the meaning of the sentence in terms of tenses.

D is only one left

My first point is that we should not confuse the use of ‘who’ with ‘whose’. ‘Who’ can be only used
for humans, while 'whose' can be used universally for humans, non-humans, and even inanimate
things.

Ex: Tom is the student who is the topper in GMAT - right , because student is a human
Tom is the student, which is the topper in GMAT - wrong because,’ which’ cannot denote
humans
Tom grows a couple of cows, who yield two liters of milk – wrong because who cannot denote
cows.
Tom grows a couple of cows, which yield two liters of milk – correct
Tom is the student, whose marks are the highest in the GMAT – right
Tom has half dozen pens, whose total is price is $5 – right again, though the pen is a not a man.
Tom grows a couple of cows, whose total yield per day is two liters of milk. – Perfectly correct,
even though cows are not humans.

All of the uses of 'whose' are correct because there is no other word to denote the possessive
form of these pronouns. Actually the possessive pronoun of ‘who’, and ‘which’ is only ‘whose’.

Second, in this case, we have to ignore - saw monkeys -; because of //ism A, B and C are out in
the dump.
Read with have, the past participle ‘seen’, is quite //. Now comes the play of logic in this
modification by the prepositional phrase. Mechanically we cannot conclude that all the eligible
nouns are vying to be modified equally; For example, a visitor is a visitor who has come to enjoy
someplace. He has not come to sleep on the branches. Nor do branches have arms and legs
hanging like socks. So it can be only monkeys. This logical perception is vital for solving such
questions in the GMAT.

That rule of yours needs a little more nuance. It's not true that "with" + [noun] +
[participle] is always wrong, 100% of the time. What''s true is that this structure is incorrect
when it contains a full action, a full action that would be more appropriately conveyed by a full
clause. For example,
With the Army of the James approaching from the west, Lee had no viable escape at the Battle
of Appomattox.
The words following "with" describe a vivid action, somebody actually doing something. For that,
we need a full clause.
Because the Army of the James was approaching from the west, Lee had no viable escape at the
Battle of Appomattox.
By contrast, the structure can be purely descriptive, lacking any connotation of some "doer"
doing something.
With an overcoat hanging over one shoulder, he saunter into the room.
That's perfectly correct. There's not an active "doer" doing something. The entire "with"
construction is purely descriptive, not conveying a separate action. That's why the structure is
100% correct in that case. Much in the same way, version (D) of the question is perfectly
correct:
Visitors to the park have often looked up into the leafy canopy and seen monkeys sleeping on
the branches, with arms and legs hanging like socks on a clothesline.
Think about this. This is not an "action." This is not a "doer" doing something. This is purely
descriptive. The "with" clause is simply painting a picture of the scene, not describing a separate
action different from the action of the main clause. In this case, the "with" structure is 100%
correct.

What matters with this structure is meaning. Meaning is always the most important thing on the
GMAT SC.
Inflation has made many Americans reevaluate their assumptions about the future; they still
expect to live better than their parents have, but not so well as they once thought they could.

(A) they still expect to live better than their parents have

(B) they still expect to live better than their parents did

(C) they still expect to live better than their parents had

(D) still expecting to live better than their parents had

(E) still expecting to live better than did their parents

Ans – B

I agree; Americans can be comparing their present living status with that of their parents until
now.
In that case we would write almost exactly what you wrote, with one change.
The word lived has to be included. "Americans still expect to live better than their parents have
lived." That sentence is correct. We must write both words, "have lived."

When verb tenses change the word "have" cannot stand alone to represent the changed tense.
In this sentence we have "live" and "have lived."

Have on its own would refer to "live" and would mean "have live." 

This sentence that uses have all by itself is not correct: Americans still expect to live better than
their parents  have.

Unfortunately, we can't create the correct sentence with our options.


lived is not available for omission, and so it cannot be "implied," either.
See the explanation below.

Further, it's true that past perfect can connote "from the past until now."
We still conjugate past perfect, though, as have  lived and present as live. We do not have
a lived that we can "imply" by writing have.

When verbs shift in a sentence, only some form of "to do" (almost always do, does, or did) will
allow us to imply the verb in a tense that is different from the one explicitly mentioned.
-- Have does not work the same way as a "to do" word does.
-- Have cannot stand in for a verb whose tense has changed.
• This question involves ellipsis and substitution.
-- In ellipsis, words that are present in the sentence are not repeated.
-- Stated differently, in elliptical construction we omit some words.
Those words are then implied in the places in which they have been omitted.
Often those omitted words will be shortened with substitution—short words that can substitute
for the omitted words.

Pure ellipsis, no verb tense shift: He will be a world-class dancer, and she will, too.  [Omitted
verb phrase: be a world-class dancer]

Ellipsis and substitution (verb tense shift): Electric cars were made to save gas, and they
do. [they = electric cars, do = save gas]

• In order to be omitted, however, the exact words first have to be present someplace in


the sentence.

On the GMAT, we must be able to insert the omitted words, exactly as they were,
and maintain correct grammar. We cannot change the words that have been omitted.

In other words, because the first verb is live, we cannot insert lived in order to use have lived.
Lived is not present in the original clause.

• Takeaway: the verb that is present "controls" what we may use in elliptical construction. The
only verb that can express changed tenses is do (does, did).

Correct: She has not  lived better than her parents have [lived].

Incorrect: She does not expect to  live better than her parents  have.
Problem: LIVED cannot be "implied by omission."
There is no "lived" to omit.

Live is present and coupled with the first subject, "they."


The second subject, "their parents," is coupled with an "elided" verb (part of the verb is
supposed to be implied).

We can attach "live" to the auxiliary verb have that is coupled with "their parents"
to see whether the auxiliary verb can stand alone.

She does not expect to  live  better than her parents  have live. 

The words do, does, and did often stand in or substitute for verb phrases.


Marisol works harder to finish her college projects than her older sister did.
(Marisol's older sister did not work as hard in college as Marisol works now.)
Andreas liked horror movies as a teenager more than 15-year-old Ricardo does.
(When Andreas was a teenager, he liked horror movies. Ricardo is a teenager who does not like
horror movies as much as Andreas liked horror movies at the same age.)

Have and will cannot stand in for verb phrases and shifted verb tenses in the same way that the
words do, does, and did can stand in.

Thus in A and B we have

A) . . . they still expect to live better than their parents have [live]

B) . . . they still expect to live better than their parents did [= lived or have lived].
Takeaways:
• if elliptical construction is an issue, be sure that the omitted words appear someplace else in
the sentence.
If a word was not in the sentence to begin with, we can't "imply" it by omission because there is
nothing to omit
• if a verb tense shift exists and ellipsis is involved, find the option that both makes the shift
clear and does so grammatically. That sentence almost certainly will use do, does, or did.
• do, does, and did are auxiliary verbs that can stand in for entire verb phrases even if the verb
tense must shift. Those words often help keep the comparison parallel.

COMMENTS

Semicolon? IF a semicolon shows up, unless the sentence is an unusual list, then the semicolon
must be between two complete independent clauses.
Two ICs can be separated by a semicolon. No connector.

Correct: Most people write with their right hand; worldwide, only 10 percent of people use their
left hand to write.

Two ICs, transitional phrase connector


The pharmacist was overworked and exhausted; as a result, she forgot to put labels on the
bottles and had to start all over again.
Other words that can come after the semicolon include:
• a coordinating conjunction (but)
• conjunctive adverbs (accordingly, thus, meanwhile, consequently)

A semicolon is needed for lists in which a comma would confuse


These contrasting colors grab attention when set next to one another: yellow and purple; red
and green; and orange and blue.

Both D and E use a semicolon that is not between two full clauses, but rather,
between (1) one subordinate clause "Still expecting to live . . . " and
(2) one full IC ("Inflation has made many Americans reevaluate their assumptions about the
future.")

• Comparison
"Than" is a comparison word. Whatever is on each side of the comparison word must be parallel.
With auxiliary (helping) verbs such as have and has, add the auxiliary onto and before the
"main" verb for the subject
whose verb has been "shortened" to an auxiliary.

If the auxiliary + main makes sense, the option is okay on the parallelism front.
In this case only option B makes sense.

The Eastern State Penitentiary was established in 1822 by reformers advocating that prisoners
be held in solitary confinement and hard labor so as to reform them.

(A) advocating that prisoners be held in solitary confinement and hard labor so as to reform
them

(B) who were advocating prisoners to be held in solitary confinement and hard labor for their
reform
(C) advocating solitary confinement and hard labor as the means to reform prisoners

(D) who advocated solitary confinement and hard labor for the means of prisoner reform

(E) advocating as the means for prisoner reform solitary confinement and hard labor

Ans - C

The Eastern State Penitentiary was established in 1822 by reformers advocating that prisoners
be held in solitary confinement and hard labor so as to reform them.

(A) advocating that prisoners be held in solitary confinement and hard labor so as to reform
them-- advocating is not a subjunctive factor

(B) who were advocating prisoners to be held in solitary confinement and hard labor for their
reform --- advocating prisoners is distortion of meaning.

(C) advocating solitary confinement and hard labor as the means to reform prisoners -- correct
choice .
(D) who advocated solitary confinement and hard labor for the means of prisoner reform --- for
the means of is unidiomatic - as the means of correct.
(E) advocating as the means for prisoner reform solitary confinement and hard labor--- 'as the
means for' prisoner reform is unidiomatic -- should 'as the means of'
Dressed as a man and using the name Robert Shurtleff, Deborah Sampson, the first woman to
draw a soldier's pension, joined the Continental Army in 1782 at the age of 22, was injured three
times, and was discharged in 1783 because she had become too ill to serve.

(A) 22, was injured three times, and was discharged in 1783 because she had become

(B) 22, was injured three times, while being discharged in 1783 because she had become

(C) 22, and was injured three times, and discharged in 1783, being

(D) 22, injured three times, and was discharged in 1783 because she was

(E) 22, having been injured three times and discharged in 1783, being

Ans – A

Quote:
(A) 22, was injured three times, and was discharged in 1783 because she had
become

I often hear people say that (A) isn’t parallel: “…Deborah Sampson… joined the Continental
Army…, was injured three times, and was discharged in 1783…” That’s actually completely
fine: we have three parallel verbs, and they make perfect sense, since they are three actions
that Sampson performed.

The objection is usually that the three verbs “aren’t in the same form” or “don’t sound the
same.” Neither of those are legitimate objections. All that really matters is that they’re three
verbs that logically are three actions performed by the subject of the clause. It’s not a problem
that one is an action verb (“joined”) and the other two are states of being (“was injured” and
“was discharged”).

The only other potential issue is the past perfect tense (“had become”) at the end of the
sentence. We can only use past perfect tense to describe an action that happened in the past,
but BEFORE some other past action, which is usually in simple past tense. And we have that: she
“had become too ill too serve” before she “was discharged.”

So let’s keep (A).

Quote:
(B) 22, was injured three times, while being discharged in 1783 because she had
become

My biggest issue is with the phrase “while being discharged in 1783.” Literally, that seems to be
saying that she simultaneously was discharged and was injured three times in 1783. And that
makes no sense – there’s no way that those things can happen at the same exact time.

Plus, I think we would need an “and” somewhere in here: there are several actions, and at least
two of them (“joined the Continental Army” and “was injured three times”) already seem to be
parallel. So the “and” is necessary.

That’s enough to eliminate (B).

Quote:
(C) 22, and was injured three times, and discharged in 1783, being

Now this is a weird mess. Sampson “joined the Continental Army… and was injured… and
discharged…” Huh? First of all, those three actions (“joined”, “was injured”, and “was
discharged”) are logically parallel to each other, so we only need one “and” – not two.

Second, it’s wrong to say that Sampson “discharged in 1783.” We could say that she WAS
discharged from the Army, or I guess we could say that she “discharged a weapon” (a semi-
obscure way to say that she fired it). But you wouldn’t just say that “Sampson… discharged.”

And “being” is also a mess. It seems to be trying to act as a modifier of some sort, but that’s
rarely an acceptable use on the GMAT. (More on “being” here.)

So we have tons of reasons to ditch (C).

Quote:
(D) 22, injured three times, and was discharged in 1783 because she was

(D) isn’t too bad, but it doesn’t make sense to say that Sampson “injured three times.” You can
say that she WAS injured three times, or maybe that she injured three enemy soldiers. But you
can’t say that she “injured three times.”

And that’s enough to disqualify (D).

Quote:
(E) 22, having been injured three times and discharged in 1783, being

Whenever you see a “having + verb” construction on the GMAT, it generally needs to be the first
of two past actions. So you could say something like “having studied all night, Souvik collapsed
on the sofa and watched three consecutive Marvel films.” In other words, he studied first, and
then collapsed. Fair enough.

But in (E), it sounds like Sampson was injured and discharged first, and THEN she joined the
Army. And that makes no sense at all.

We also have a problem with “being”, which is apparently being used as a modifier in (E) – and
that’s a use that we almost never see on correct GMAT questions. “Being” isn’t always wrong on
the GMAT, but it’s probably wrong here.
But even if you really love “being” in (E) for some inexplicable reason, we still have lots of
reasons to eliminate (E). And we’re left with (A).

irst of all i will say that verb-ed form is either acts as a verb or as a modifier

verb-ed -->this will act as a verb when subject itself does this (verb-ed thing) action ex: alex
played well ....here played is acting as verb
verb-ed--->this will act as a modifier when this (verb-ed thing) is not done by the subject.

now coming to question


here the subject is Deborah Sampson
in the non underlined part it is written Deborah Sampson joined......here "joined" acts as a verb
in the underlined part it is written Deborah Sampson...was injured .....,and was discharged....so
its list kind of thing x,y,and z therefore x,y,z should be parallel
..now----- joined ,was injured ,and was discharged are all verb so its parallel....
note if in place of was injured....only injured is written and in place of " was discharged "only
discharged is written then in that case discharged and injured act as modifier as in option
C,D,E.....HENCE it is not parallel to JOINED .....so it is wrong

now choosing answer


(A) 22, was injured three times, and was discharged in 1783 because she had become(parallel
so correct)
(B) 22, was injured three times, while being discharged in 1783 because she had
become( meaning is wrong ..in this it means she was injured and discharged
simultaneously...which is not the intended meaning)
(C) 22 and was injured three times, and discharged in 1783, being(...here joined ,was injured
are verb while discharged is modifier so non parallel therefore incorrect)
(D) 22, injured three times, and was discharged in 1783 because she was(again injured is not
parallel.....so incorrect)
(E) 22, having been injured three times and discharged in 1783, being( againg discharged is
modifier and not verb ,...and therefore not parallel....also it means having been injured and
discharged ..both actions are occuring together which is not correct.

In his experiments with gravity, Isaac Newton showed how the motion of each planet in the
solar system results from the combined gravitational pull of the Sun and of all the other planets,
each contributing according to their mass and distance from the others.

(A) of all the other planets, each contributing according to their


(B) of all the other planets, with each of them contributing according to their
(C) all the other planets, each of which contributing according to its
(D) all the other planets, each contributing according to its
(E) all the other planets, each of which contribute according to their

Ans – D

Notice that each answer choice has “each” that makes the subject singular. Even if “each” is
referring to “Sun and all the other planets”, it turns all the entities into singular. Hence, when we
use “each” to talk about “Sun and all the other planets”, the pronoun that we use to refer to
those entities that actually stands for “each” should be singular.
So in this problem, we have a case of pronoun antecedent number disagreement and not the
ambiguity. We can eliminate Choices A, B, and E because of the use of plural “their” with
singular “each”.

Choice C is incorrect because “which” should introduce a clause. There is no verb after “which”
that makes the choice incorrect.

Choice D is precise and free of errors.

C. all the other planets, each of which contributing according to its

Use of 'each of which' is imprecise here. 'each of which' like 'which' requires a verb after that. As
there is no verb after 'each of which', the construction is imprecise.

Correct use of 'each of which' -


select any two answer choices,  each of which can  be used to complete the sentence.
I have bought several books,  each of which is  based on a different theme.

D. all the other planets, each contributing according to its

Yes, "their" is wrong because it doesn't agree with "each."

You can't say "each of which contributing." Adding "of which" makes "each" into a subject which
would then need the verb "contributes."

In D, "each contributing" serves as a modifier--"each" doesn't serve as a subject.

Compare these:

The lobby was full of aspiring actors, each dreaming of landing a big role.
The lobby was full of aspiring actors, each of whom dreamed of landing a big role.

Those both work. What we wouldn't want to say is this:

The lobby was full of aspiring actors, each of whom dreaming of landing a big role.

It's not that a solitary "each" can't serve as a subject--it's that it doesn't have to. The word
"each" can serve as either a pronoun or a modifier (adjective/adverb).

Pronoun: Each one of these paintings is worth a million dollars.


Modifier: There are three bedrooms, each with its own bathroom.

However, we can also use "each" as a pronoun in a modifying clause. In this case, we use "of
which/of whom" to signal that we are building such a clause.

He ordered several different dishes, each of which could have served as a meal in its own right.
Here, "each" is the subject and "could" is the main verb. We typically do this to express
something more complicated than we could in a normal modifier. Notice that because we have a
complex set of verbs ("could have served"), it would be hard to express this without a full
clause. If we said "each serving as a meal in its own right," that would change the intended
meaning (each dish actually is a meal).

what is the role of 'contributing and the phrase connected with it..
It basically speaks on the previous clause, thus modifying the clause..

But use of each of which makes 'each of which' as a SUBJECT and therefore requires a VERB..
It would be correct if we say--
each of which is contributing..
each of which contributes...

Each can be a pronoun or an adjective..


when used as an adjective / modifier, it is not necessary that it should have a verb, as in this Q
in correct choice

Each of which is always a subject/pronoun and will be a part of a clause with a VERB..

Scientists have identified an asteroid, 2000 BF19, that is about half a mile wide and, if it strikes
Earth, it can do tremendous damage to part of the planet but probably not cause planetwide
destruction.

A. and, if it strikes Earth, it can do tremendous damage to part of the planet but

B. and, if it would strike Earth, part of the planet could experience a tremendous amount of
damage but it would

C. and that, if it were to strike Earth, could do tremendous damage to part of the planet but
would

D. and that, if Earth is struck by it, can do part of the planet tremendous damage, but it would

E. and that, if it strikes Earth, it could experience a tremendous amount of damage but

Ans - C

First recollect the IF... THEN .. structure for the following cases:

1. IF simple present THEN simple future.


2. IF hypothetical subjunctive THEN conditional (would / could etc.).

Now consider a sentence in the first structure:


If it strikes Earth, it will probably not cause planet-wide destruction.

How would you convert this sentence to a future event not likely to happen (i.e., the second
structure):
If it struck Earth (OR if it were to strike Earth), it would probably not cause planet-wide
destruction.

Note that "would" in the THEN clause is a grammatical requirement to match the IF clause.
Therefore in this case "would probably" is not considered redundant.

In the previous example, there is no IF.. THEN.. structure to justify the use of "would". In such
standard cases, "will probably" is correct.

A. and, if it strikes Earth, it can do tremendous damage to part of the planet but -- it can refer
back to earh or asteroid.. not clear..Hence, incorrect.

B. and, if it would strike Earth, part of the planet could experience a tremendous amount of
damage but it would -- it can refer to part of the planet, asteroid or earth.. not
clear..Hence, incorrect.

C. and that, if it were to strike Earth, could do tremendous damage to part of the planet but
would -- correct..subjunctive mood for hypothetical situations..

D. and that, if earth struck by it can do part of the planet tremendous damage, but it would --
Passive..seems like earth is causing the damage.Hence, incorrect.

E. and that, if it strikes earth, it could experience a tremendous amount of damage but --Who


will exprience damage ? earth or asteroid..not clear...hence, incorrect.

Faced with an estimated $2 billion budget gap, the city’s mayor proposed a nearly 17 percent
reduction in the amount allocated the previous year to maintain the city’s major cultural
institutions and to subsidize hundreds of local arts groups.

(A) proposed a nearly 17 percent reduction in the amount allocated the previous year to
maintain the city’s major cultural institutions and to subsidize

(B) proposed a reduction from the previous year of nearly 17 percent in the amount it was
allocating to maintain the city’s major cultural institutions and for subsidizing

(C) proposed to reduce, by nearly 17 percent, the amount from the previous year that was
allocated for the maintenance of the city’s major cultural institutions and to subsidize

(D) has proposed a reduction from the previous year of nearly 17 percent of the amount it was
allocating for maintaining the city’s major cultural institutions, and to subsidize

(E) was proposing that the amount they were allocating be reduced by nearly 17 percent from
the previous year for maintaining the city’s major cultural institutions and for the subsidization

Ans - A
The question reads - Faced with an estimated $2 billion budget gap, the city's
mayor proposed a nearly 17 percent reduction in the amount allocated the previous
year to maintain the city's major cultural institutions and to subsidize hundreds of
local arts groups.

This is the correct answer - A is the correct choice.

Here 'proposed' is the verb in the clause 'faced with .....mayor proposed...'

My question is since 'proposed' is a command subjunctive verb, why isn't it being


used in the format 'mayor proposed that ....'?

Any help in understanding command subjunctive better is much appreciated.

Hi abm03,

Please note that there is no verb in the part of the sentence after “proposed”.

• Faced with an estimated $2 billion budget gap, the city's mayor proposed a nearly 17
percent reduction in the amount allocated the previous year

o to maintain the city's major cultural institutions and


o to subsidize hundreds of local arts groups.

“allocated” in the highlighted entity is a verb-ed modifier. Since allocated is not a verb, it


cannot be written in subjunctive form.

Since allocated is not in subjunctive form, the above sentence stands correct even without the
usage of “that”.

Also note that “propose” is not the subjunctive verb. “propose” requires a subjunctive verb after
it i.e. the next verb should be in subjunctive form.

Note that the following is an example of subjunctive usage.

a. The parliament members demanded that the President resign immediately.

In this sentence “resign” is the verb in subjunctive form.

Notice that there is no verb after “proposed” in the OG sentence you presented. So the noun
phrase “a nearly 17 percent reduction in the amount” is the object to the verb “proposed”.
“allocated…” is the verbed modifier, adding more information about the “amount”.
In 1776 Adam Smith wrote that it is young people who have “the contempt of risk and the
presumptuous hope of success” needed to found new businesses.

(A) who have


(B) with
(C) having
(D) who are those with
(E) who are the ones to have

Ans – A

Wow! What a brilliant question that purely hinges on understanding the meaning and ability to
realize that "needed" is used as a past participle (and not a verb) here.

n 1776  Adam Smith  wrote that  it  is young people  who  have “the contempt of risk and the
presumptuous hope of success” needed to found new businesses .

Let's first understand the meaning of this sentence. The sentence intends to say that Adam
Smith wrote something in 1776. What did he write?

He wrote that young people possess two qualities (if I may): “the contempt of risk and the
presumptuous hope of success”. What is the significance of these two qualities.

These two qualities are required to found = establish new businesses.

Now let's come to the grammar part of it.

In the above-mentioned sentence, the subjects are highlighted in blue while the verbs are in
green. All the subjects have appropriate verbs.

Please note that the word needed is a verb-ed modifier that further explains “the contempt of
risk and the presumptuous hope of success”.

We do not need anymore verbs in the sentence as there are no subjects with missing verbs.

We may expand this sentence as In 1776  Adam Smith  wrote  that  it  is  young
people  who  have  “the contempt of risk and the presumptuous hope of success”  that  are
needed to found new businesses.

The added that clause is just the expanded version of the original sentence. We actually derive
verb-ed modifiers in this way by removing the subject and the helping verb.

You can read our very famous article named ED FORMS - Verbs or Modifiers to learn how to
distinguish between a verb-ed modifier and a simple past tense verb in the following link:
https://gmatclub.com/forum/ed-forms-verbs-or-modifiers-134691.html

A. In 1776 Adam Smith wrote that

it is young people who have “the contempt of risk and the presumptuous hope of success”
needed(required) to found (establish) new businesses. This is a relative clause with the modifier
phrase starting with 'who' modifying young people.
B. In 1776, Adam Smith wrote that

it is young people with “the contempt of risk and the presumptuous hope of success” needed to
found new businesses. --- This prepositional modifier 'with' modifying young people lacks a verb
to complete the meaning.

Discussion of greenhouse effects has usually focused on whether the Earth would warm and by
how much, but climatologists have indicated all along that the most obvious effects, and those
that would have the largest impact on people, would be extremes of temperature, precipitation,
and storminess.

(A) the most obvious effects, and those that would have the largest impact on people, would be
extremes of temperature, precipitation, and storminess

(B) the effects that are the most obvious ones, extremes of temperature, precipitation, and
storminess, would be those impacting the most on people

(C) those effects to have the largest impact on people, extremes of temperature, precipitation,
and storminess, are what are the most obvious effects

(D) extremes of temperature, precipitation, and storminess, the most obvious effects, that they
would have the largest impact on people

(E) extremes of temperature, precipitation, and storminess, which are the most obvious effects,
are those to impact the most on people

Ans - A

In this case, I am rather skeptical about the use of the modal would rather than will. Would Is
normally used
1. when a direct speech that is set in the past is converted into a reported or indirect speech,
where in the term will is turned into ther past would, to avoid the shift of tense.

2. As a past subjunctive, when presenting a hypothetical ness that is never going to happen.

Ex: if I were to choose to visit a planet, I would rather choose Jupiter, because it is the biggest.

3. Where you want to impart a certain degree of speculation about the event or the
phenomenon

Ex: With Sachin losing his sheen, I think that Mumbai Indians would be hammered out in this
IPL.
Well, it is just a crude guess and it may or may not happen. There may be other instances, but
by and large, thee are the uses of the auxiliary verb would.
In this case, the climatologists have all along indicated that the most obvious effects ware those
three things. In fact, it sounds as if these are the only three things that are sure to happen.
There seems to be not even an iota of speculation about it. How can we use a verb such as
would in such cock-sure circumstances. I rather think that the use of will be the most
appropriate in the given text and in A

The new image of Stone Age people as systematic hunters of large animals, rather than merely
scavenging for meat, have emerged from the examination from the examination of tools found in
Germany, including three wooden spears that archaeologists believe to be above 400,000 years
old.

A. merely scavenging for meat, have emerged from the examination from the examination of
tools found in Germany, including
B. as mere scavenging for meat, have emerged from examining tools found in Germany, which
include
C. as mere meat scavengers, has emerged from examining tools found in Germany that includes
D. mere scavengers of meat, has emerged from the examination of tools found in Germany,
which includes
E. mere scavengers of meat, has emerged from the examination of tools found in Germany,
including

Ans – E

Answer to your first question: Yes, it is alright to not repeat “as” after “rather than” because it is
implied or understood. “As” already appears once in the sentence. Another thing that we need to
note in Choice C is parallelism. In the parallel list, the first entity is “hunters of large animals”
and the second one is “meat scavengers”. Now, it is not always necessary for the entities in the
parallel list to be absolutely parallel. However, in this case, it is possible. We can write “meat
scavengers” as “scavengers of meat” that will make the entities absolutely parallel. So go for it.

Answer to your second question: Yes, “examining tools” have a little problem. Here “examining”
is now an adjective that is modifying “tools”, suggesting that the “tools” are used for examining
things. It no longer conveys that the new image has emerged from the examination of the tool.

In 1850 Lucretia Mott published her Discourse on Women, arguing in a treatise for women to
have equal political and legal rights and for changes in the married women’s property laws.

A. arguing in a treatise for women to have equal political and legal rights
B. arguing in a treatise for equal political and legal rights for women
C. a treatise that advocates women’s equal political and legal rights
D. a treatise advocating women’s equal political and legal rights
E. a treatise that argued for equal political and legal rights for women
Ans - E

A> The construction is “arguing”+”in a treatise…” or participle + prepositional phrase. Or simply,


it’s a participial phrase. Now, the participial phrase most likely modifies the subject, which is
“Lucretia Mott”. Now the question is - Is Lucretia Mott arguing or is the argument in her book?
When we look at the phrase “arguing in a treatise” , it is apparent that Lucretia is not arguing.
Also, to+be construction is wordy. Be watchful

B> Same as above. The participial phrase as a modifier is not doing the justice.

C> An appositive “a treatise that..” is fixing the above problem. But it is introducing another
problem. “a treatise that advocates women’s equal political and legal rights and for changes in
the married women's property laws”. It is important to understand the construction of
appositive. An appositive could be simply noun or noun+modifier. Here, we can write - a treatise
(noun) that that advocates women’s equal political and legal rights (relative or adjective clause)
and (a treatise) for changes in the married women's property laws (prepositional phrase). As you
can see, a treatise for changes in the married women's property laws (prepositional phrase) does
not make any sense. To correct this, we need (a treatise) that advocates for changes in the
married women's property laws (adjective clause)
D> It is having the same problem as C. A treatise for changes in the married women's property
laws (prepositional phrase) does not make any sense.
E> This answer choice fixed both problem. It introduced appositive correctly. Also, a parallelism
is established with two prepositional phrases “for equal political and legal rights for women” and
“for changes in the married women's property laws” that are modifying verb “argue” and act as a
adverbial modifier

Almost like clones in their similarity to one another, the cheetah species’ homogeneity makes
them especially vulnerable to disease.

(A) the cheetah species’ homogeneity makes them especially vulnerable to disease

(B) the cheetah species is especially vulnerable to disease because of its homogeneity

(C) the homogeneity of the cheetah species makes it especially vulnerable to disease

(D) homogeneity makes members of the cheetah species especially vulnerable to disease

(E) members of the cheetah species are especially vulnerable to disease because of their
homogeneity

Ans – E

Much of the action is in the non-underlined portion of this sentence: “Almost like clones in their
similarity to one another...” That phrase includes the plural pronoun “their”, so it needs to be
followed by a plural noun that could reasonably be described as “like clones” in terms of their
similarity to each other.

With that in mind…

Quote:
(A) the cheetah species’ homogeneity makes them especially vulnerable to disease

(A) definitely doesn’t work, because “the cheetah species’ homogeneity” is singular, and it’s
unreasonable to say that the “homogeneity” is “almost like clones in their similarity to one
another.”

So (A) is out.

Quote:
(B) the cheetah species is especially vulnerable to disease because of its
homogeneity

The trouble here is that “the cheetah species” is singular, and once again, a singular species
can’t logically be described as “almost like clones in their similarity to one another.” (And in case
you’re wondering: yes, “species” can also be plural, but in this case, it’s followed by “is”, and
then “its” refers back to “species” later in the sentence. So “species” is definitely singular in this
particular sentence.)

We can eliminate (B), too.

Quote:
(C) the homogeneity of the cheetah species makes it especially vulnerable to disease

(C) repeats the exact same error as (A): “the homogeneity” is singular, and it’s still totally
unreasonable to say that the “homogeneity” is “almost like clones in their similarity to one
another.”

That takes care of (C).

Quote:
(D) homogeneity makes members of the cheetah species especially vulnerable to
disease

(D) rearranges a few things, but the heart of the problem is still completely the same as in (A)
and (C): “homogeneity” is singular, and it’s once again unreasonable to say that the
“homogeneity” is “almost like clones in their similarity to one another.”

(D) is gone, and I really hope we like (E), or else we’ll be starting over…

Quote:
(E) members of the cheetah species are especially vulnerable to disease because of
their homogeneity

Yup, this is fine! “Members” is plural, and it makes sense to say that “members of the cheetah
species” are “almost like clones in their similarity to one another.”

“Their” also jumps out at me at the end of the sentence, and that’s fine, too: “their” refers to
“members of the cheetah species.”
Over the next few years, increasing demands on the Chattahoochee River, which flows into the
Apalachicola River, could alter the saline content of Apalachicola Bay, which would rob the
oysters there of their flavor, and to make them decrease in size, less distinctive, and less in
demand.

(A) which would rob the oysters there of their flavor, and to make them decrease in size,
(B) and it would rob the oysters there of their flavor, make them smaller,
(C) and rob the oysters there of their flavor, making them decrease in size,
(D) robbing the oysters there of their flavor and making them smaller,
(E) robbing the oysters there of their flavor, and making them decrease in size,

Ans – D

(A) which would rob the oysters there of their flavor, and to make them decrease in
size,

The first thing that jumps out at me is the underlined “which” modifier. I don’t think that it
makes a whole lot of sense: “Apalachicola Bay” certainly doesn’t “rob oysters there of their
flavor”, and neither does “the saline content of Apalachicola Bay.” The alteration of the saline
content robs oysters of their flavor – but that’s a verb here (“could alter”), and “which” generally
doesn’t modify a verb on the GMAT.

The parallelism is also a huge problem here. The phrase “to make” follows the “and”, so we’d
need another infinitive verb earlier in the sentence. But I don’t see anything that could possibly
work.

So we can eliminate (A).

Quote:
(B) and it would rob the oysters there of their flavor, make them smaller,

Hopefully, the word “it” jumps out at you whenever you see it. You’re looking for a singular
referent, but in this case, I don’t see a lot of great options: we have the saline content,
Apalachicola Bay, or a couple of different rivers, but none of those are really performing the
action of robbing oysters of their flavor. It’s the alteration of the saline content – caused by
increasing demands on the Chattahoochee River – that rob the oysters of their flavor. So the
pronoun “it” is wrong.

Plus, we have some funky parallelism stuff going on here: “rob the oysters there of their flavor,
make them smaller, less distinctive, and less in demand.” I’d be OK if there was an “and” before
“make”: that way, “smaller”, “less distinctive”, and “less in demand” could all be parallel to each
other.

But in this case, the list makes no sense: it’s a hodgepodge of verbs (“rob” and “make”) and
modifiers (“less distinctive” and “less in demand”). (B) is definitely out.

Quote:
(C) and rob the oysters there of their flavor, making them decrease in size,

This one is tricky! It looks like “rob” is parallel to the verb “could alter”, and I guess that’s OK:
“… increasing demands on the Chattahoochee River… could alter the saline content…
and rob oysters there of their flavor…” That doesn’t sound too bad, but we could argue that the
alteration of the saline content is the thing that robs the oysters of their flavor, not the
“increasing demands on the river” – so the two verbs “rob” and “could alter” probably shouldn’t
be parallel to each other. That’s awfully subtle, and you shouldn’t feel badly if you didn’t notice
that there’s a problem with it.

The other issue with (C) is the parallelism at the end of the sentence: “making them decrease in
size, less distinctive, and less in demand.” So “less in demand” and “less distinctive” are both
modifiers. Fair enough. But then “decrease in size” is a verb phrase, which can’t be parallel to
those two modifiers.

Nasty stuff, in my opinion. (C) is gone.

Quote:
(D) robbing the oysters there of their flavor and making them smaller,

This sounds weird. “Robbing the oysters there of their flavor and making them smaller…” Hm,
that’s a mouthful. Say it five times fast, and you probably won’t want to choose it as your
answer.

But it’s right. “Robbing the oysters there of their flavor” is now a modifier, giving us more
information about the entire previous clause about increasing demands on the river, and
alterations of the saline content. That makes perfect sense: the entire, long-winded situation –
beginning with the “increasing demands” on the river – robs oysters of their flavor, so the “-ing”
modifier is perfect.

And the parallelism at the end of the sentence is great, too: “making them smaller, less
distinctive, and less in demand.” Three parallel modifiers, all describing what happens to the
oysters.

Let’s keep (D).

Quote:
(E) robbing the oysters there of their flavor, and making them decrease in size,

The comma after “flavor” is a minor issue. In general, the GMAT doesn’t spend a lot of time
testing us on the subtleties of comma usage, but there’s no real need for the comma here, since
“robbing” and “making” are very nicely parallel with each other. Don’t lose sleep over this, since
it’s rarely – if ever – a deciding factor on these questions.

The bigger issue is the parallelism error at the end of the sentence: just as in (C), “decrease in
size” isn’t parallel to “less distinctive” and “less in demand,” since “decrease” is a verb. And that’s
the best reason to eliminate (E), and settle for (D).

First discovered more than 30 years ago, Lina's sunbird, a four-and-a-half-inch animal found in
the Philippines and that resembles a hummingbird, has shimmering metallic colors on its head; a
brilliant orange patch, bordered with red tufts, in the center of its breast; and a red eye.

(A) found in the Philippines and that resembles

(B) found in the Philippines and that, resembling

(C) found in the Philippines and resembling

(D) that is found in the Philippines and it resembles

(E) that is found in the Philippines, resembling

Ans – C

(A) found in the Philippines and that resembles


(B) found in the Philippines and that, resembling

We can ditch (A) and (B) pretty quickly: "and" is followed by a phrase beginning with "that" --
and there's nothing earlier in the sentence that could possibly be parallel to the phrase beginning
with "that."

Quote:
(C) found in the Philippines and resembling

(C) doesn't SOUND parallel, but it is!

"Found" is just an adjective that modifies the animal, and so is "resembling." Sure, they
don't sound the same, but they're both adjectives, so we're all good here. Let's keep (C).

(And for more on parallelism and "-ing" words, check out this
article: https://gmatclub.com/forum/experts-topi ... 39780.html).

Quote:
(D) that is found in the Philippines and it resembles

The parallelism isn't quite right in (D), either. The phrase that follows "and" is a brand-new
clause: "it resembles a hummingbird." And that can't possibly be correct here, since "it
resembles a hummingbird" isn't parallel to "that is found in the Philippines". Besides, the
underlined portion is part of a modifier, and I have no idea why we would stick a full,
independent clause inside that modifier. So (D) is gone.

So we're down to (C) and (E). Here's the sentence again, with (E) punched in:
Quote:
(E) First discovered more than 30 years ago, Lina's sunbird, a four-and-a-half-inch
animal that is found in the Philippines, resembling a hummingbird, has shimmering
metallic colors on its head...

The trouble here is that we have a messy (and unnecessary) "double modifier": "that is found in
the Philippines" modifies the animal, and so does the phrase "resembling a hummingbird." Notice
that (C) puts these two modifiers in a nice, parallel structure, with an "and" separating the two
modifiers ("found..." and "resembling..."). That isn't happening in (E) -- it's almost as if
"resembling a hummingbird" is trying to modify the other modifier, "that is found in the
Philippines," and that just doesn't make any sense.

(C) is much clearer, and it's our winner.

The answer can as well be given in a little more complex way: This problem is a perfect example
of: Participles.
Small Review:
A participle is a verbal (verb form) which is used as an adjective.
Generally,there are three participles that are commonly used as adjective:
1. the present participle(active voice); the past participle (passive voice) and the perfect
participle (active voice). * The present participle always ends in ing *
2. The past participle usually ends in ed, d, t, n, or en.
3. The past participle of some of the verbs do not have distinctive endings: swum, gone, sung
etc. *The perfect participle is always formed by prefixing the word having to the past participle:
having sung, having driven, having seen etc

In the given sentence, we can clearly see that for the 1st participial phrase, the action
of "found" happened in the past because Lina's sunbird was found more than 30 years
ago. Further Lina's sunbird did not do any finding instead they were found by X not mentioned in
the sentence so passive voice is right.

The second participial phrase starting with "resembling" implies that the "Lina's sunbird" is
still resembling in the present (a present condition), so the use of present participle is better. We
cannot say that the Lina's sunbird "resembled" because then this would illogically mean that they
no longer resemble the humming bird.

First discovered more than 30 years ago, (-modifier-) Lina's sunbird (-


subject-), a four-and-a-half-inch animal found in the Philippines and that
resembles a hummingbird (-modifier-),has(-verb-) shimmering metallic colors
on its head ;a brilliant orange patch, bordered with red tufts, in the center of its
breast; and a red eye. (-list of things-)

In late 1997, the chambers inside the pyramid of the Pharaoh Menkaure at Giza were closed to
visitors for cleaning and repair due to moisture exhaled by tourists, which raised its humidity to
such levels so that salt from the stone was crystallizing and fungus was growing on the walls.

(A) due to moisture exhaled by tourists, which raised its humidity to such levels so that salt from
the stone was crystallizing

(B) due to moisture that tourists had exhaled, thereby raising its humidity to such levels that salt
from the stone would crystallize

(C) because tourists were exhaling moisture, which had raised the humidity within them to levels
such that salt from the stone would crystallize

(D) because of moisture that was exhaled by tourists raising the humidity within them to levels
so high as to make the salt from the stone crystallize

(E) because moisture exhaled by tourists had raised the humidity within them to such levels that
salt from the stone was crystallizing

Ans – E
As per the OG explanation, 'them' in C and D seems to refer to 'tourists', but in E
'them' clearly refers to chambers.
Can someone please explain the concept of the pronoun reference in this case?

What I understood is that:


in the case of E tourists is the noun in the modifier 'exhaled by tourists'. So, 'tourists'
loose the significance as it is out of scope for the remaining main sentence/clause.
But is is the right creteria to reject the pronoun reference to noun 'tourists'.

I wonder what you mean by saying tourists loose the significance as it is out of scope
coming back to the pronoun reference issue -- I think the reason THEM refers to tourists in C
and D and not in E
is that in C and D there is an intervening Clause which introduces TOURISTS in such a way that
it can act as a more proximate antecedent (than chambers); recall that a pronoun refers to
the nearest logical antecedent

In C -- it says -- Tourists WERE exhaling moisture ---- hence TOURISTS can act as a subject in
its own right.

In D -- it says -- because of moisture that was exhaled by tourists -- again here even though the
subject of the relative clause introduced by THAT is MOISTURE yet THEM cannot refer to
moisture but instead refers to the predicate of the clause (Tourists) in this case.

In E if you notice because moisture exhaled by tourists cannot position tourist as a likely


antecedent (because 'tourists' is part of a phrase and hence cannot stand on its own as a).

In E : "exhaled by tourists" is a modifier that is modifying moisture and so the sentence


"moisture had raised the humidity within them" is the sentence and so it is more clear that
"them" refers to chambers.

In D: "tourists raising the humidity within them" "raising the humidity with in them" modifies
"tourists". Hence, it looks like "them" refers to "tourists".

Not sure of the case with C.

Doubt 2:
In option A, idiom "such levels so that" is used. Is this correctly used ?

Doubt 3:
It is mentioned that "due to" is used imprecisely to express causal relation. Can you please
eloborate on how "due to" can be used effectively to express these relations?

Option C is a total disaster in that :


i) because + verb-ing form is incorrect on GMAT
ii) "them" may refer to "tourists".
iii) usage of "such that" implies intention, nowhere mentioned in the intended meaning. Hence
incorrect.

Doubt 2: "such levels so that"- incorrect because of two issues:


i) such levels implies that we have already talked about some levels and here mean to refer to
those levels only, but nowhere we have talked about some levels.
ii) so that implies intention, which is incorrect here.

Doubt 3: Yes "due to" is used to show a causal relation. Whenever you see "due to" just replace
it with "caused by" and then analyze whether there is any causal relation or not. If there is then
"due to" is fine.
The tourism commission has conducted surveys of hotels in the most popular resorts, with the
ultimate goal of reducing the guests who end up expressing overall dissatisfaction with the
service in the hotels.

(A) with the ultimate goal of reducing the guests who end up expressing overall dissatisfaction
with the service in the hotels

(B) with the goal to ultimately reduce the number of guests who end up expressing overall
dissatisfaction with the hotels’ service

(C) ultimately with the goal to reduce expressions of overall dissatisfaction by the guests with
the hotel service

(D) in an ultimate attempt to reduce the number of guests that ends up expressing overall
dissatisfaction with the hotels’ service

(E) with the ultimate goal of reducing the number of guests who express overall dissatisfaction
with the hotels’ service

Ans – E

A) with the ultimate goal of reducing the guests who end up expressing overall dissatisfaction
with the service in the hotels -> The number of guests can be reduced. The guests
themselves cannot be reduced. On a funny note, you cannot really reduce their
height or something :D So, eliminate.

B) with the goal to ultimately reduce the number of guests who end up expressing overall
dissatisfaction with the hotels’ service -> Incorrect modifier 'ultimately'. This modifier
should modify 'goal' and not 'reduce'. So, eliminate.

C) ultimately with the goal to reduce expressions of overall dissatisfaction by the guests with the
hotel service -> 'goal to reduce expressions' totally distorts the meaning. So, eliminate.
D) in an ultimate attempt to reduce the number of guests that ends up expressing overall
dissatisfaction with the hotels’ service -> 'that' cannot be used to refer to people (i.e.
guests). So, eliminate.

E) with the ultimate goal of reducing the number of guests who express overall dissatisfaction
with the hotels’ service -> By POE, this is the right answer. Though I am not very
convinced about the usage of 'with' here. Going by the rule, 'with' should modify
resorts here. Resorts don't have any ultimate goal.

A long-term study of some 1,000 physicians indicates that the more coffee these doctors drank,
the more they had a likelihood of coronary disease.

(A) more they had a likelihood of coronary disease

(B) more was their likelihood of having coronary disease

(C) more they would have a likelihood to have coronary disease

(D) greater was their likelihood of having coronary disease

(E) greater was coronary disease likely

Ans – D

The positive....comparative...superlative forms of certain adjectives of quantity are as follows:

set 1. many...more...most
set 2. much...more....most
set 3. great...greater...greatest

While set 1 (many...more...most) is used for countable nouns, set 2 (much...more....most) and
set 3 (great...greater...greatest) are used for uncountable nouns.

Now there could be 2 types of uncountable nouns


type a. Quantity word / numbers themselves (e.g. price, volume, weight, probability etc.) are
uncountable.
type b. Other uncountable nouns (honesty, poverty, etc as you mentioned)

The set 2 (much...more....most) is used for type b (other uncountable nouns) uncountable
nouns.
The set 3 ( great...greater...greatest) is used for type a (Quantity word / numbers) uncountable
nouns.
Therefore we see that "more" can be used for (i) countable nouns and (ii) uncountable nouns
that are not quantity words/ numbers.
"Greater" can be used for (iii) uncountable nouns that are quantity words/ numbers.

"the more coffee these doctors drank, more they had a likelihood of coronary disease

As the second part of the structure refers to 'likelihood', we need 'greater'. 'greater' is used when
the noun is a number. For example: percentage, rate, likelihood, distance and population.

Also, 'likelihood' means 'the probability of a specified outcome'. Thus, it's imprecise to say they
had a likelihood/probability of coronary disease. The correct usage should be "likelihood of
having coronary disease'.

E goes out as it distorts the meaning. 'likelihood' was greater, NOT the disease.

This leaves us with D.

The use of lie detectors is based on the assumption that lying produces emotional reactions in an
individual that, in turn, create unconscious physiological responses.

(A) that, in turn, create unconscious physiological responses

(B) that creates unconscious physiological responses in turn

(C) creating, in turn, unconscious physiological responses

(D) to create, in turn, physiological responses that are unconscious

(E) who creates unconscious physiological responses in turn

Ans - A

The easiest place to start is probably with the create/creates split. What "create(s)...responses"?
The emotional reaction. (Strictly speaking the subject of the verb "create" is the pronoun "that."
More on this issue later.) Since "emotional reactions" is plural, we want a plural verb, "create."
That eliminates B and E. "Creating" in C is a present participle, and "to create" in D is an
infinitive, so neither of these is either singular or plural.

Next we might consider which form of the verb "create" is appropriate, the verb "create," the
present participle "creating," or the infinitive "to create." A verb-form split usually signals either
an idiom problem or a problem with parallelism, but not in this case. Rather, neither C nor D
captures the original meaning. Answer C: A present participle (-ing word) immediately preceded
by a noun (no comma separating them) attributes action to that noun. So C means that "an
individual" creates responses. Answer D: Here, "to create" suggests that the goal of producing
emotional reactions is to create physiological responses.

That leaves A, the correct answer.

Addendum 1. I began by asking "What creates responses?" This common-sense question is


usually adequate to match a verb to its subject. Sometimes you'll have a couple of reasonable
answers, and you'll have to look more closely at the grammar. The verbs in relative clauses--
noun-modifying clauses that start with "that" or one of the "wh-" words, "which," "who/m,"
"when," "where"--actually modify the relative pronouns that precede them, but those relative
pronouns are themselves singular or plural depending on the nouns to which they refer. The
long and short of it is that you're probably better off with the common-sense question than with
the subtle grammatical question here.

Addendum 2. With few exceptions, relative clauses modify the nouns or noun phrases they
touch. In this question, "that...create...responses" touches the noun "individual," but it also
touches the noun phrase "emotional reactions in an individual," that is, the noun "reactions" plus
the adjective "emotional" plus the prepositional phrase "in an individual." That whole phrase, or
if you prefer the noun at its heart, is modified by the relative clause. Frankly, I think that it's a
little clumsy here, and I might have had a bit of trouble had I started with the modifier issue.

that is used as a relative pronoun here. Since it is a pronoun, it can only refer to nouns.
So, that cannot refer to lying produces emotional reactions in an individual, because lying
produces emotional reactions in an individual is a clause since it has a verb produces; as I
mentioned, that can only refer to nouns/noun-phrases, not to verbs/clauses.

Also, notice that that cannot refer to individual, because when used as a relative


pronoun, that cannot refer to people (in this case individual). So basically, that refers
to emotional reactions.

Here are some genuine examples of ‘explained as’ and ‘explained by’

The Tajmahal is explained as a monument of love and caring of a husband for his wife.

A Himalayan blunder can be explained as a blunder as great or tall as the Himalayas.

GMAT can be explained as a test of endurance and application

Examples for ‘explained by’

The cascading downfalls in the history of stock markets can be only explained by the investors’
bandwagon psychology

The reason behind the FIFA muddle can be explained by the venality some depraved souls.

Ryunosuke Akutagawa‘s knowledge of the literatures of Europe, China, and that of Japan were
instrumental in his development as a writer, informing his literary style as much as the content
of his fiction.

(A) that of Japan were instrumental in his development as a writer, informing his literary style as
much as

(B) that of Japan was instrumental in his development as a writer, and it informed his literary
style as well as

(C) Japan was instrumental in his development as a writer, informing both his literary style and
(D) Japan was instrumental in his development as a writer, as it informed his literary style as
much as

(E) Japan were instrumental in his development as a writer, informing both his literary style in
addition to

Ans – C

First, it can refer to either knowledge or development. In both cases, it does not fit in logically.

Second, the knowledge was the first factor, his development as a writer is the next factor, and
the third is the information about his style and function. The development and the information
thereof are independent functions and not cause and effects. D is distorting the meaning by
implying that development and information occurred simultaneously or the information occurred
because of development. ‘As’ has meaning of ‘because ’ and ‘at the same time’. This is fatal
error.

Now, coming to this sentence, Ryunosuke Akutagawa’s knowledge informed the following two
things:

1. Literary style
2. Content of his fiction

With D, the sentence would be:

Ryunosuke Akutagawa‘s knowledge of the literatures of Europe, China and  Japan was
instrumental in his development as a writer, as it informed his literary style  as much
as the content of his fiction .

Now, here is the tricky thing. The above sentence can be interpreted as:

Ryunosuke Akutagawa‘s knowledge of the literatures of Europe, China and Japan was
instrumental in his development as a writer, as it informed his literary style as much as the
content of his fiction (informed his literary style)

Which means Ryunosuke Akutagawa’s literary style was informed by the following:

1. Ryunosuke Akutagawa’s knowledge


2. Content of his fiction

So, there is an ambiguity of meaning, if D is the answer choice. Now, perhaps this ambiguity
might be “tolerated” in some sentence in GMAT, if there is no better option; but here, C is
definitely clearer.

(1) If you use "less," "fewer," or "more" to make a comparison, and if both nouns/prepositional
phrases/whatever being compared appear after the "less" or "more," then you must use "than"
to introduce the second. (This eliminates A, B, and D)

He has fewer cats than dogs.


This is correct, because "fewer" precedes the first noun, "cats," and "than" precedes the second
noun, "dogs."
This is the structure you can expect to see on the GMAT.
He has three dogs, but fewer cats.
This is correct. It uses "fewer" without "than," but one of the nouns being compared appears
before the "fewer."
Offhand, I don't remember seeing this structure on the GMAT.

(2) All of the words "less," "fewer," and "more" imply comparison, so any additional comparison
signal, such as "compared to" or "relative to" is redundant. (This eliminates A, B, and D)

(3) Whatever the elements being compared, they must be both logically comparable and
structurally similar. No comparison signal/parallel marker can coordinate two elements unless
those elements are the same part of speech. (This eliminates A, B, and D)

The computer company’s present troubles are a result of technological stagnation, marketing
missteps, and managerial blunders so that several attempts to revise corporate strategies have
failed to correct it.

(A) so that several attempts to revise corporate strategies have failed to correct it

(B) so that several attempts at revising corporate strategies have failed to correct

(C) in that several attempts at revising corporate strategies have failed to correct them

(D) that several attempts to revise corporate strategies have failed to correct

(E) that several attempts at revising corporate strategies have failed to correct them

Ans – D

NOTE: In A, B, and C, the phrases "so that" and "in that" would modify some kind of action, as
in "He went to the store so that he could find Amy" or "It was a bad move in that it resulted in
him getting dumped by Amy." We don't have an action we want to modify here (i.e., the only
verb we have in the first half of the sentence is "are", and we're probably not modifying that).

NOTE: D & E both correctly employ a relative pronoun to modify the final noun. The phrase
beginning with "that" (a relative pronoun) thus modifies whatever word came before it. In this
case, this final phrase must be modifying "managerial blunders", not the entirety of the list. For
this reason, the "them" is majorly wrong. First, it is ambiguous (could refer to any individual
plural noun in the list, or the whole list). Second, it is nonsense, because we're already in a
modifying phrase, so there should be no need to point towards the noun we're talking about.

Yes, relative pronouns are bit wishy-washy this way. They can modify the entire noun phrase,
but they can also just modify the last noun. Logic really dictates what's happening.

I am in love with my car and the girlfriend that I met last week.
Clearly "that I met last week" is not modifying both nouns, but only the last one. Think of the
modifier as a kind of adjective.

I am in love with the time and money that this new job affords me.

Clearly "that this new job affords me" is modifying both "time" and "money".

Either of these is considered correct, as long as it's clear which is meant.

Das könnte Ihnen auch gefallen